obmnivel3b

56
SISTEMA ANGLO DE ENSINO 1 ANGLO VESTIBULARES I. MEDIATRIZ DE UM SEGMENTO Seja AB um segmento de uma reta. Mediatriz de AB é o lugar de pontos do plano eqüidistantes de A e B. Na figura ao lado a reta r é a mediatriz de AB . II. UM TRIÂNGULO 1. Elementos Ângulo interno de um triângulo é o ângulo determinado por dois lados. Ângulo externo é o ângulo formado por um dos lados do triângulo e pelo prolongamento do lado adjacente (ao lado). Altura é um segmento da reta perpendicular traçada de um vértice à reta suporte do lado oposto que tem por extremidades esse vértice e o ponto de encontro com a reta suporte. Mediana é um segmento de reta em que as extremidades são um vértice e o ponto médio do lado oposto a este vértice. Bissetriz interna é um segmento de reta que une um vértice a um ponto do lado oposto dividindo o ângulo interno deste vértice ao “meio”. Mediatriz de um lado é a reta perpendicular a este lado pelo seu ponto médio. Notações: Usaremos nestas aulas: ABC para se referir a medida do ângulo A ˆ BC em uma certa unidade. Quando B é um dos vértices de um triângulo ABC, usaremos ˆ B, ou simplesmente B, para se referir a medida do ângulo A ˆ BC em uma certa unidade. a,b,c comprimentos dos lados opostos aos vértices (ângulos) A, B e C respectivamente. O segmento CS é a bissetriz interna relativa ao vértice C. S é o desta bissetriz O segmento CH é uma altura relativa ao vértice C. H é o pé da perpendicular (projeção ortogonal) de C sobre a reta suporte do lado AB. CÂB, (Â ou A) ângulo inter- no relativo ao vertice A . •C ˆ BD ângulo externo relativo a B. CAB (ou A) medida de A. CM é uma mediana relativa ao vértice C A B S C A C H B A M B C D A B r anglo SISTEMA DE ENSINO Aulas 1 e 2 Prof. Ponce / Roberto NÍVEL 3, SEGUNDA FASE, GEOMETRIA

Transcript of obmnivel3b

Page 1: obmnivel3b

SISTEMA ANGLO DE ENSINO 1 ANGLO VESTIBULARES

I. MEDIATRIZ DE UM SEGMENTO

Seja AB um segmento de uma reta.Mediatriz de AB é o lugar de pontos do plano eqüidistantes de A e B.Na figura ao lado a reta r é a mediatriz de AB .

II. UM TRIÂNGULO1. Elementos

Ângulo interno de um triângulo é o ângulo determinado por dois lados.Ângulo externo é o ângulo formado por um dos lados do triângulo e pelo prolongamento do lado adjacente (ao lado).Altura é um segmento da reta perpendicular traçada de um vértice à reta suporte do lado oposto que tem por

extremidades esse vértice e o ponto de encontro com a reta suporte.Mediana é um segmento de reta em que as extremidades são um vértice e o ponto médio do lado oposto a este

vértice. Bissetriz interna é um segmento de reta que une um vértice a um ponto do lado oposto dividindo o ângulo

interno deste vértice ao “meio”.Mediatriz de um lado é a reta perpendicular a este lado pelo seu ponto médio.

Notações:

Usaremos nestas aulas:• ∠ ABC para se referir a medida do ângulo ABC em uma certa unidade.

Quando B é um dos vértices de um triângulo ABC, usaremos B, ou simplesmente B, para se referir a medida doângulo ABC em uma certa unidade.

• a,b,c comprimentos dos lados opostos aos vértices (ângulos) A, B e C respectivamente.

• O segmento CS é a bissetrizinterna relativa ao vértice C.

• S é o pé desta bissetriz

• O segmento CH é uma alturarelativa ao vértice C.

• H é o pé da perpendicular(projeção ortogonal) de Csobre a reta suporte do lado AB.

• CÂB, (Â ou A) ângulo inter-no relativo ao vertice A .

• CBD ângulo externo relativoa B.

• ∠ CAB (ou ∠ A) medida de A.• CM é uma mediana relativa

ao vértice C

A BS

C

A

C

H BA M B

C

D

A

B

r

angloSISTEMA DE ENSINO Aulas 1 e 2

Prof. Ponce / Roberto

NÍVEL 3, SEGUNDA FASE, GEOMETRIA

Page 2: obmnivel3b

SISTEMA ANGLO DE ENSINO 2 ANGLO VESTIBULARES

2. Classificação de Triângulo Relativa aos seus ÂngulosSeja ABC um triângulo

ABC é isósceles se, e somente se, dois de seus lados (ângulos) são congruentes. (I)ABC é eqüilátero se, e somente se os 3 seus lados (ângulos) são congruentes. (II)ABC é retângulo em B se, e somente se, um de seus ângulos interno (B) é reto. (III)ABC é acutângulo se, e somente se, seus ângulos internos são agudos. (I) e (II)ABC é obtusângulo se, e somente se, um de seus ângulos internos é obtuso. (IV)

III. RELAÇÕES ENTRE DOIS TRIÂNGULOS1. Congruência entre dois Triângulos

Dois triângulos são congruentes se, e somente se, existir entre eles uma das seguintes combinações de ângu-los congruentes e lados congruentes.

(LLL) três lados, (LAL) dois lados e o ângulo compreendido entre eles.(ALA) dois ângulos e o lado compreendido entre eles.(LAA0) dois ângulos e um lado oposto a um destes ângulos.

Dois lados e um ângulo não compreendido entre eles reto ou obtuso.(se o ângulo é agudo, os triângulos podem não ser congruentes)Para expressar que o triângulo ABC é congruente ao triângulo DEF usaremos ∆ABC ≅ ∆ DEF

2. Semelhança entre dois TriângulosDois triângulos são semelhantes se, e somente se, existir entre eles uma das seguintes combinações de ân-

gulos congruentes e lados proporcionais:(LLL) três lados, (AA) dois ângulos. (LAL) dois lados e o ângulo compreendido entre eles.Dois lados e um ângulo não compreendido entre eles reto ou obtuso.(se o ângulo é agudo, os triângulos podem não ser semelhantes)Para dizer que o triângulo ABC é semelhante ao triângulo DEF usaremos ∆ABC ∼ ∆ DEF

IV. POLÍGONOS1. Região poligonal fechada é uma figura plana limitada por segmentos de retas consecutivos e não colineares

(que se fecham).

A,B,C, ....................... são vértices consecutivos do polígonoAB, BC, CD, DE ......... são lados consecutivos do polígono.ABC, BCD, CDE, .......... são ângulos internos consecutivos.A’BC, B’CD, C’DE, ........ são ângulos externos consecutivos.

Exemplos

Triângulo: polígono de 3 lados.

Quadrilátero: polígono de quatro lados.

Pentágono: polígono de cinco lados.

Hexágono: polígono de 6 lados.

A

B C

A

B C

A

B C

A

B C(I) (II) (III) (IV)

POLÍGONO

A’

B’C

D

E

F

A

B

C’

D’E’

F’

Page 3: obmnivel3b

SISTEMA ANGLO DE ENSINO 3 ANGLO VESTIBULARES

2. A reunião destes segmentos é denominada de Polígono. Estes segmentos são denominados de lados do polígono enquantoque os seus extremos são denominados de vértices do polígono.

3. Os segmentos cujos os extremos são dois vértices não consecutivos des-te polígono são denominados de diagonal.

4. Um polígono é convexo se, e somente se, sua região poligonal fe-chada contém todo segmento de reta cujas as extremidades são pon-tos internos desta região. Na figura ao lado ABCDEF é um polígonoconvexo de 6 lados (hexágono). O segmento FD é uma das diagonaisdeste polígono.

5. Um polígono não convexo é denominado de côncavo. Na figura ABCD éum polígono côncavo de 4 lados (quadrilátero), enquanto que o quadrilá-tero LMNP é convexo.

• AC é uma de suas diagonais. Observe que esta diagonal nãoestá contida na região poligonal deste polígono.

6. Um polígono é regular se, e somente se, os seus lados são con-gruentes e seus ângulos internos também congruentes.

V. CIRCUNFERÊNCIA1. Circunferência de centro O e raio r (r � 0) é o lugar geométrico

dos pontos P de um plano tais que OP— = r. Círculo de centro O e raior é o lugar de todos os pontos do plano limitados por uma circunferên-cia de centro O e raio r.

2. Três pontos não colineares determinam uma circunferência.

3. Sejam A, B e P pontos distintos de uma circunferência λ de centro O eraio r, denomina-se:• O segmento AB de corda de λ. Quando O pertence a AB dizemos

que AB é um diâmetro de λ.

• arcos AB, de λ aos conjuntos de todos os pontos de λ que estão“compreendidos” entre os pontos A e B, em que um deles contém Pe o outro não. Para se referir ao arco AB que contém P usaremos a

simbologia APB. (figura 1)

4. A medida de qualquer um destes arcos é definida como a razão docomprimento do arco AB para o comprimento do raio de λ, na mesma uni-dade.

5. Ângulo Central é o ângulo cujo vértice é o centro O.Na figura 1, o ângulo AÔB é o ângulo central.

6. Ângulo inscrito é o ângulo formado por duas cordas distintas mascom uma extremidade comum. Na figura 1, APB é um ângulo inscrito.

7. A medida de um ângulo central é a medida do arco (correspon-dente) da circunferência compreendido por seus lados.

Propriedades:1. Existe uma única circunferência que “passa” por três pontos não coli-

neares.2. Duas cordas de uma mesma circunferência tem comprimentos iguais se,

e somente se, distam igualmente do centro. (figura 4)

C

D

AB

L

MN

P

A

B

P

O

(Figura 1)

A

t

O

(Figura 2)

A

B

P

O

(Figura 3)

θ

C

D

E

F

A

B

REGIÃOPOLIGONAL

Page 4: obmnivel3b

SISTEMA ANGLO DE ENSINO 4 ANGLO VESTIBULARES

3. Se os lados dos ângulos APB (inscrito) e AÔB (central) determinam omesmo arco na circunferência λ, então a medida do ângulo centralé o dobro da medida do ângulo inscrito. (figura 3)

4. Todo diâmetro divide a circunferência em dois arcos de medidas iguais(semicircunferências)

5. Duas cordas de mesmo comprimento em uma mesma circunferênciadeterminam arcos de mesma medida. (figura 4)

6. Uma reta t é tangente a λ em um ponto A se, e somente se, o raio AOdesta circunferência é perpendicular a t em A. Nestas condições, A édenominado de ponto de tangência. (figura 2)

7. (Potência de um ponto P em relação a uma circunferência)• Se AB e CD são duas cordas de um circulo que concorrem em P (ou

seus prolongamentos), então PA— ⋅ PB— = PC— ⋅ PD— , (figura 5)

ReciprocamenteQuatro pontos A, B, C, D pertencem a circunferência de um circulo se,PA— ⋅ PB— = PC— ⋅ PD— , onde P é a intersecção das retas AB e CD.• Se P é um ponto externo a um circulo e T, A, B são pontos perten-

centes a circunferência deste circulo tais que PT é uma reta tangen-te e PAB uma secante, então PT—2 = PA— ⋅ PB— (figura 6)

ReciprocamenteSe três pontos P, A, B são colineares, com P não entre os pontos A eB, e T um ponto da circunferência não pertencente a reta PAB tais quePT—2 = PA— ⋅ PB—, então PT é tangente a circunferência ABT.

VI. TEOREMAS (PROPRIEDADES):Nos teoremas seguintes ABC é um triângulo com lados de comprimentos a, b e c opostos respectivamente aos

vértices A, B e C.

T1. (Desigualdade Triangular)Um triângulo ABC existe se, e somente se, a � b + c, b � a + c e c � a + b

T2. Oposto ao maior lado de um triângulo está o ângulo de maior medida evice-versa.

T3. (Propriedade do ângulo externo do triângulo)Cada ângulo externo é igual a soma dos dois ângulos internos e não adja-centes a ele.

T4. Soma dos ângulos internos é 180º.

T5. A soma dos ângulos externos é 360º.

T6. (Triângulo Retângulo)

ABC é um triângulo retângulo em A se,e só se, a2 = b2 + c2 (Pitágoras)

Propriedades do triângulo retângulo.• A circunferência que passa pelos pontos A, B e C e tem BC como diâmetro. O ponto médio de BC como cen-

tro e o segmento AO mediana relativa a hipotenusa BC) como raio. vide (figura 7).• (Relações Métricas no triângulo retângulo) (figura 8)

Se ABC é um triângulo retângulo em A, H pé da perpendicular da altura relativa ao vértice A, então

(RELAÇÕES MÉTRICAS)

BA BH BC

CA CH CB

HA HB HC

AH BC AB AC

2

2

2

=

=

=

=

⋅⋅ ⋅

14

42

44

3

A

C

B

DP

(Figura 5)

(Figura 6)

AB

T

P

A

B CO

OA = OB = OC = BC12

(Figura 7)

OH

B

A Cb

c

a

(Figura 8)

d

dO

M

L

P

N

(Figura 4)

Page 5: obmnivel3b

SISTEMA ANGLO DE ENSINO 5 ANGLO VESTIBULARES

• Conseqüências importantes

T7. Triângulo ABC qualquer

• (LEI DOS COSSENOS)

• (LEI DOS SENOS)

(R é o raio da circunferência circunscrita ao triângulo ABC)

T8. (STEWART)Se ABC é um triângulo e P um ponto do lado BC distinto de B e C, demodo que BP

—= m e PC

—= n, então

a ⋅ (p2 + m ⋅ n) = b2 ⋅ m + c2 ⋅ n

(Uma justificativa para o resultado acima, pode ser obtida pelo o teo-rema dos cossenos e o fato da soma de cossenos:

cos ∠ APB + cos ∠ APC

ser igual a zero)

T9. Área de um Triângulo (Quadrado)Indicando a área do triângulo ABC por (ABC), tem-se que:

Onde ha, hb e hc são respectivamente os comprimentos das alturasdeste triângulo relativas aos lados (bases) de comprimentos a,b e c.Equivalentemente temos também:

(ABC) = ⋅ bcsen = ⋅ acsenB= ⋅ a ⋅ b ⋅ senC

* Dois triângulos de bases BC e CD e alturas relativa a estas basesde mesmo comprimento h, conforme figura. A razão entre as áreasdestes dois triângulos é igual a razão entre suas bases, isto é,

* A razão entre as áreas de dois triângulos semelhantes é o quadrado da razão de semelhança.* A área de um quadrado de lado L é L2.

( )( )

.ABCACD

BCCD

=

12

12

12

( )ABC ah ah aha b c= = =⋅ ⋅ ⋅1

212

12

asen A

bsenB

csenC

R= = = 2

a b c bc A

b a c ac B

c a b ab C

2 2 2

2 2 2

2 2 2

2

2

2

= +

= +

= +

⋅ ⋅⋅ ⋅⋅ ⋅

– cos

– cos

– cos

AB

AC

HBHC

2

2=

1 1 12 2 2

AH AB AC= +

A

C

Bc

b a

RO

A

B CP

a

bc

p

nm

A

B C

hbc

a

A

B D

h

C

Page 6: obmnivel3b

SISTEMA ANGLO DE ENSINO 6 ANGLO VESTIBULARES

T10. Paralelas e Tansversais (Tales)Se um conjunto de três ou mais retas paralelas distintas de um pla-no são “cortadas” por duas retas (transversais), então a razão de doissegmentos quaisquer de uma destas transversais é igual a razão dossegmentos correspondentes da outra.

Conseqüência Importante (BASE MÉDIA)Uma reta é paralela a um dos lados de um triângulo se, e somente se,ela divide esses dois lados em segmentos proporcionais.O segmento MN cujos extremos são os pontos médios dos lados AB eAC de um triângulo ABC é paralelo ao lado BC e é a metade deste,

isto é, O segmento MN é denominado de base média

do triângulo ABC.

T11. Bissetrizes Interna e Externa• Bissetriz Interna

P é um ponto interno do lado BC. AP é bissetriz interna relativa ao vértice A se, e somente se,

• Bissetriz ExternaQ é um ponto do prolongamento do lado BC. AQ é bissetriz externa relativa ao vértice A se, e somente se,

Propriedade (bissetriz Interna ): É o lugar dos pontos eqüidistantes dos lados do ângulo

VII. EXEMPLOS1. (Olimpíada Canadense) A altura de um triângulo relativa ao vértice A intercepta o lado BC em D. Uma circun-

ferência tangencia BC em D, intercepta AB em M e N e intercepta AC em P e Q.

Prove que

ResoluçãoUsando o teorema de Pitágoras nos triângulos retângulos ADB e ADCe em seguida a propriedade de potência de um ponto em relação a umacircunferência, podemos escrever da figura acima:• Pitágoras no ∆ADB e potência de B.

AD—2 = AB

—2 – BD— 2 = AB

—2 – BN—– ⋅ BM

—–

= AB—2 – (AB

—– AN

—–) ⋅ (AB

—– AM

—–)

AD—2 = AB

— ⋅ (AM—–

+ AN—–

) – AM—– ⋅ AN

—–

AM AN

AC

AP AQ

AB

+ = +

P

A

E

F

SB C

AS é bissetriz

PE = PF

BQ

QC

AB

AC=

BP

PC

AB

AC=

MN

BC=2

.

A

B

C C’

B’

A’

B

M N

A

C

ABBC

= A’B’B’C’

(Base média)

A

B P C

A

B QC

A

MP

Q

CDB

N

Page 7: obmnivel3b

SISTEMA ANGLO DE ENSINO 7 ANGLO VESTIBULARES

• Raciocinando de análogo, agora com o ∆ADC e o ponto e C, obtemos:

AD—2 = AC

— ⋅ (AP—

+ AQ—–

) – AP— ⋅ AQ

—–

Desde que AM—– ⋅ AN

—–= (AP

—+ AQ

—–) (potência de A ),

temos AB— ⋅ (AM

—–+ AN

—–) = AC

— ⋅ (AP—

+ AQ—–

),

Daí, o que finaliza a demonstração.

2. (Olimpíada Espanhola) Seja P um ponto no interior de um triângulo ABC, de modo que no triângulo ABP, tem-se: AP = BP.Sobre cada um dos outros lados de ABC se constrói exteriormente triângulos BQC e CRA, ambos semelhantes aotriângulo ABP, mas com BQ = QC e CR = RA. Provar que os pontos P, Q, C e R ou estão alinhados, ou são vérti-ces de um paralelogramo.

ResoluçãoOs triângulos ABC e PBQ são semelhantes pelo caso LAL.

pois ∠ ABC = ∠ PBQ e ,

. De modo análogo, ABC é semelhante a APR. Conseqüentemente, por transiti-

vidade, temos que PBQ e APR são também semelhantes (onde) PB— = PA—).

Em particular: ∠ ARP = ∠ ACB e ∠ BQP = ∠ ACB.

Por outro lado, fazendo ∠ BAP = ∠ ABP = α resulta

∠ QPR = 360º – (180º – 2 ⋅ α) – (A + B)= 180º + 2 ⋅ α (180º – ∠ ACB) = 2 ⋅ α + ∠ ACB

∠ QCR = ∠ ACB + 2 ⋅ α∠ PRC = (180º – 2 ⋅ α) – ∠ ARP = 180º – 2 ⋅ α – ∠ ACB∠ PQC = (180º – 2 ⋅ α) – ∠ BQP = 180º – 2⋅ α – ∠ ACB

Estas igualdades, implicam que os pares de ângulos opostos do quadrilátero PQCR são congruentes.

Assim, se 180º – 2 ⋅ α – ∠ ACB = 0, isto é, , tem-se que P, Q, C e R estão alinhados.

Caso contrário, isto é, , tem-se que P, Q, C e R são vértices de um paralelogramo.

➲ atividades de aula1. (Olimpíada Espanhola) ABCD é um quadrado de lado 1. E e F pontos sobre os lados DC e CB respectivamente,

distintos dos vértices do quadrado e tais que: ∠ EAF = 45º. Determine o perímetro do triângulo CEF.

2. a) (Olimpíada Francesa) Prove que: Se ABCD é um quadrilátero convexo de área S, entãoAB—

+ BC—–

+ CD—–

+ DA—–

� AC—

+ BD—–

b) (Olimpíada de Leningrado) Prove que é impossível construir a estrela abaixo, de modo que ocorra simultanea-mente as desigualdades:

AB—

� BC—–

CD—–

� DE—–

EH—–

� FG—–

GH—–

� HI—

IK—

� KA—–

α ≠ ∠1802

º – ACB

α = ∠1802

º – ACB

BP

BQ

ca

=

AM AN

AC

AP AQ

AB

+ = +

A

B

QC

R

Pa

b

c

A

B C

D

E F

G

HK

I

Page 8: obmnivel3b

SISTEMA ANGLO DE ENSINO 8 ANGLO VESTIBULARES

3. (Olimpíada Colombiana)a) ABCDEFGH é um octógono regular inscrito em uma circunferência de diâmetro 1. Sendo P um ponto do menor

arco AH desta circunferência, determine o valor da soma:

PA—2 + PB

—2 + PC—2 + PD

—–2 + PE—2 + PF

—2 + PG—–2 + PH

—–2

b) A1A2A3A4.........An é um polígono regular com número n par de lados inscrito em uma circunferência de diâmetro 1.P um ponto qualquer desta circunferência distinto dos vértices deste polígono. Calcule a soma S dada por:

4. (OBM) Sobre os lados AB e AC de um triângulo acutângulo ABC são construídos, exteriormente ao triângulo,semicírculos tendo estes lados como diâmetros. As retas contendo as alturas relativas aos lados AB e AC cortamesses semicírculos nos pontos P e Q. Prove que AP = AQ.

5. Seja ABC um triângulo qualquer. Prove que:

6. a) (Olimpíada regional da África do Sul) Utilizando o teorema dos Cossenos, prove o teorema de STEWART:Se ABC é um triângulo qualquer, P um ponto do lado BC distinto de B e C, de modo que AP

—= p, BP

—–= m e

PC—–

= n, então a ⋅ (p2 + m ⋅ n) = b2 ⋅ m + c2 ⋅ n (vide figura):

b) (Treinamento para a Olimpíada Canadense) Seja P um ponto interno do triângulo ABC tal que ∠ PAC = 10º,∠ PCA = 20º, ∠ PAB = 30º e ∠ ABC = 40º. Utilizando o Teorema dos Senos, determine, em graus, ∠ BPC.

➲ atividades de casa1. (Olimpíada Regional Francesa)

a) ABCD é um quadrado de lado l.Dado que AE = x � l, CF = y � l, ∠ EDF = ∠ FDC e DE = a.Determine o valor de x + y em função de a

b) Determine, em graus,o valor de S:

S = u + v + w

A B

D C

E

F

y

x

A

B PCm n

bcp

a

sen A sen B sen C senB senC A

sen B sen C sen A senC sen A B

sen C sen A sen B sen A senB C

2 2 2

2 2 2

2 2 2

2

2

2

= +

= +

= +

⋅ ⋅ ⋅⋅ ⋅ ⋅⋅ ⋅ ⋅

– cos

– cos

– cos

14

24

3

S PAi

i

n=

=∑

2

1

u

v

w

Page 9: obmnivel3b

SISTEMA ANGLO DE ENSINO 9 ANGLO VESTIBULARES

2. a) (Treinamento para Olimpíada Colombiana) ABC é um triângulo. D é um ponto do lado BC tal que D = 2 e DC = 1.Sabendo que: ∠ ACD = 45º e ∠ ADB = 60º. Determine, em graus, a medida do ângulo ABC.

b) (Olimpíada da África do Sul) Seja ABC um triângulo isósceles de base BC e ∠ BAC = 100º. A bissetriz do ânguloBÂC intercepta AC em P. Prove que BC

—–= AP

—+ PB

—–

c) (Treinamento para Olimpíada Canadense) Na figura abaixo, ∠ QAB = ∠ QAC = 10º, ∠ QBA = 20º e ∠ QBC = 100º.Calcule, em graus, a medida do ângulo ACQ.

3. a) (Olimpíada de Maio) ABC é um triângulo retângulo de hipotenusa BC. Seja S um ponto desta hipotenusa e M oponto médio do cateto AB. Dado que AC = 1, ∠ SAB = 45º e AS ⊥ SM, determine o comprimento do cateto AB.

b) (Olimpíada Regional Argentina) Em um triângulo ABC retângulo em A traçam-se as bissetrizes internas BB´ eCC´. Sabendo-se que AB´= 1 e AC´= 1, calcular o comprimento da hipotenusa.

c) (Olimpíada Regional Mexicana) Na figura, ABC é um triângulo eqüilátero de lado 3, e a reta PA é paralela a retaBC. Determine o comprimento do segmento CS, sabendo que PQ—– = QR—– = RS—–

4. (OBM) Sejam ABCD um quadrado, M o ponto médio de AD e E um ponto sobre o lado AB. P é a interseção de ECe MB. Mostre que a reta DP divide o segmento EB em dois segmentos de mesma medida.

5. a) (Olimpíada Espanhola) Prove que: Se ABCD é um quadrilátero convexo, então

b) (Olimpíada Espanhola) Prove que: A soma dos comprimentos de todos os lados e diagonais de um quadrilátero

convexo, não é menor que

6. (Olimpíada de Leningrado) Quadrados ABDE e BCFG são construídos externamente a um triângulo ABC. Proveque o triângulo ABC é isósceles, se DG é paralela a AC.

2 2 2( ).+

1 2� �

AB BC CD DA

AC BC

+ + ++

A

B C S

R

Q

P

A

B

B’C

xC’

1

1

A

C

MB

S

10°10°

20° 100°

A

B

Q

C

Page 10: obmnivel3b

SISTEMA ANGLO DE ENSINO 10 ANGLO VESTIBULARES

7. Usando o teorema de Stewart, mostre que:Se ABC é um triângulo qualquer e AP é mediana relativa ao lado BC talque AP

—= p, então

(Este resultado é conhecido por Teorema de Apolônio)

LEITURA COMPLEMENTARResolvendo Problemas de Geometria através da Área de Triângulos

O conceito da área de um triângulo, muitas vezes, é um grande aliado na resolução de problemas de geometriaplana. Um exemplo disso, você pode verificar nos exemplos abaixo.

1. (Olimpíada Americana) ABCDE é um pentágono regular. AP, AQ eAR são perpendiculares traçadas de A até o lado DC, prolonga-mento de CB e o prolongamento de DE respectivamente, comoindicado na figura ao lado. Sabendo que O é centro da circunfe-rência circunscrita ao pentágono e OP

—–= 1, determine o valor da

soma AO—–

+ AQ—–

+ AR—–

ResoluçãoInicialmente, observe que a região pentagonal limitada pelo pentágono regular ABCDE pode ser dividida em 5 regiõestriangulares, limitadas por triângulos isósceles e congruentes ao triângulo ODC.

Então, a área do pentágono ABCDE, é dada por [1]

Por outro lado, traçando as diagonais AD e AC, dividimos a região pentagonal ABCDE em três regiões triangulares,limitadas agora pelos triângulos: ADE, ACD e ABC. Nestas condições, a área do pentágono é dada agora pela so-ma destas três áreas. Daí,

Portanto, de [1] e [2], podemos escrever:

como AB—

= BC—

= CD—

= DE—

, pois ABCDE é um pentágono regular, segue-se da igualdade acima, queAR—–

+ 1 + AO—–

+ AQ—–

= 5. Portanto, AO—–

+ AQ—–

+ AR—–

= 4.

Resposta: 4Nota: (ABCDE) e (XYZ) representam respectivamente a área de um pentágono ABCDE e área de um triângulo XYZ.

2. (Olimpíada Australiana) Sejam X, Y e Z pontos dos lados BC, CA e AB de um triângulo ABC de modo que ossegmentos AX, BY e CZ concorram no mesmo ponto P interior ao triângulo conforme figura abaixo.

Prove que: PXAX

PYBY

PZCZ

+ + =1

A

B CX

P

YZ

DE AR DC AO BC AQ DC⋅ + ⋅ + + ⋅ = ⋅ ⋅2

12 2

51

2( )

( ) ( ) ( ) ( )

( )[ ]ABCDE ABC ACD ADE

DE AR DC AO BC AQ= + + = ⋅ + ⋅ + + ⋅2

12 2

2

51

2⋅ ⋅DC

b c pa2 2 2

2

22

+ = +

⋅B

A

PC

bc

a

m n

p

A

R

E

D P C

B

Q

1

O

Page 11: obmnivel3b

SISTEMA ANGLO DE ENSINO 11 ANGLO VESTIBULARES

Resolução“As áreas de triângulos de alturas de comprimentos iguais são proporcionais aos comprimentos das bases destestriângulos”. Assim, como os triângulos:

• XBP e XBA, têm a mesma altura com respeito a reta AX, Então: [1]

• XCP e XCA, têm a mesma altura com respeito a reta AX, Então: [2]

Nestas condições, de [1] e [2], segue-se que: . Raciocinando de modo análogo

ao que foi feito acima, encontramos:

Portanto,

• (MNP) indica a área de um triângulo de vértices M, N e P.

3. P é um ponto interno do lado BC.Se AP é bissetriz interna relativa ao vértice A, então

(Teorema da Bissetriz Interna)

Demonstração

Os triângulos PBA e PCA com respeito a reta BC tem a mesma altura. Então: [1]

Desde que P pertence a bissetriz BP. P é eqüidistante dos lados AB e AC. Então, o ∆PBA com respeito a reta AB

tem a mesma altura que o ∆PCA com respeito a reta AC. Daí, [2]

Portanto, de [1] e [2], conclui-se que: BP

PC

AB

AC=

AB

AC

BPACPA

= ( )( )

BPPC

BPACPA

= ( )( )

BP

PC

AB

AC=

PXAX

PYBY

PZCZ

BPCABC

CPAABC

APBABC

ABCABC

+ + = + + = =( )( )

( )( )

( )( )

( )( )

1

PYBY

CPAABC

e PZCZ

APBABC

= =( )( )

( )( )

PXAX

XBP XCPXBA XCA

BPCABC

= ++

=( ) ( )( ) ( )

( )( )

PX

AX

XCPXCA

= ( )( )

,

PX

AX

XBPXBA

= ( )( )

,

A

B P C

Page 12: obmnivel3b

SISTEMA ANGLO DE ENSINO 12 ANGLO VESTIBULARES

I. CEVIANA1. Definição

Ceviana (*) de um triângulo é um segmento unindo um vértice do triân-gulo a um ponto qualquer do lado oposto. Assim, se X, Y e Z são pontossobre os lados BC, CA e AB respectivamente do triângulo ABC, então ossegmentos AX, BY e CZ são cevianas, conforme a figura ao lado:

2. Cevianas Notáveis de um TriânguloA altura, mediana e bissetrizes são exemplos de cevianas do triângulo ABC. Recordemos abaixo a definição delas.Altura é um segmento da reta perpendicular traçada de um vértice à reta suporte do lado oposto que tem por ex-

tremidades esse vértice e o ponto de encontro com a reta suporte.Mediana é um segmento de reta em que as extremidades são um vértice e o ponto médio do lado oposto a este

vértice.Bissetriz interna é um segmento de reta que une um vértice a um ponto do lado oposto dividindo o ângulo

interno deste vértice ao “meio”.

(*) a palavra ceviana é decorrente do nome do matemático italiano Giovanni Ceva (1648-1734) que publicou em 1678 os teoremas seguintes:

II. TEOREMAST1. (CEVA) Se as três cevianas AD, BE, CF de um triângulo ABC, são

concorrentes, então

Demonstração:

Seja P o ponto comum as três cevianas.Sejam R e S respectivamente as intersecções das semi-retas BE e CFcom a reta construída paralelamente a BC por A, conforme figura aolado. Então por semelhança de triângulos temos:

[1] (∆AFS ~ ∆BFC)

[2] (∆BPD ~ ∆RPA e ∆DPC ~ ∆APS) e

[3] (∆CEB ~ ∆AER)

Agora, multiplicando membro a membro, os resultados obtidos em [1], [2] e [3] encontramos:

AF

FB

BD

DC

CE

EA

SA

BC

AR

SA

BC

AR⋅ ⋅ ⋅ ⋅= = 1

CE

EA

BC

AR=

BD

DC

AR

SA=

AF

FB

SA

BC=

AF

FB

BD

DC

CE

EA⋅ ⋅ = 1

Aulas 3 e 4NÍVEL 3, SEGUNDA FASE, GEOMETRIA

A

Y

CXB

Z

A

B CD

EF

P

A

B CD

EF

P

S R

Page 13: obmnivel3b

SISTEMA ANGLO DE ENSINO 13 ANGLO VESTIBULARES

T2. (CEVA — Recíproco de T1) Se as três cevianas AF, BE, CF de um triângulo ABC são tais que ,

então as três cevianas são concorrentes (tem um único ponto em comum).

Demonstração:Seja P o ponto comum de duas das três cevianas, por exemplo, deBE e CF, o que não perde em generalidade.Suponhamos D’ um ponto sobre BC de modo que a ceviana AD´passe por P.Nestas condições, decorre do teorema T1, que:

[1]

Por outro lado, por hipótese, temos

[2]

Assim, de (1) e (2), segue-se que:

Daí conclui-se que D’ coincide D, o que significa dizer que AD passa por P, intersecção das outras duas ce-vianas. Consequentemente as 3 cevianas AD, BE, CF são concorrentes em P.

T3. (Menelaus de Alexandria (aproximadamente 100A.C.)) Sejam três pontos D, E e F respectivamente sobre os la-dos BC, CA e AB (ou seus prolongamentos) de um triângulo ABC.

D, E e F são colineares se, e somente se,

1. Dizemos que D, E e F são colineares se pertencem a uma reta. Esta reta frequentemente é denominada detransversal ou secante e referiremos a ela simplesmente por DEF.

2. Necessariamente um dos pontos D, E ou F pertencem ao prolongamento do lado.

Demonstração:Traça-se por B uma paralela ao lado AC. Sendo X a interseção desta paralela com a reta DEF, segue-se que ospares de triângulos abaixo são semelhantes (DBX, DCE) e (BXF, AEF)

Desta semelhança, podemos escrever:

Combinado estas igualdades resulta

Portanto, AF

FB

BD

DC

CE

EA⋅ ⋅ = 1

BD

DCCE

FB

AFEA⋅ ⋅=

BD

DC

BX

CE

BX

EA

FB

AFe= =

AF

FB

BD

DC

CE

EA⋅ ⋅ = 1

A

B CD

X

F

E

A

B CD

X

FE

BD

D C

BD

DC

'

'=

AF

FB

BD

DC

CE

EA⋅ ⋅ = 1

AF

FB

BD

D C

CE

EA⋅ ⋅ ='

'1

AF

FB

BD

DC

CE

EA⋅ ⋅ = 1

A

B CD’

EF

P

D

Page 14: obmnivel3b

SISTEMA ANGLO DE ENSINO 14 ANGLO VESTIBULARES

Reciprocamente: Sejam D, E, F pontos sobre as retas suporte BC, CA e AB dos lados do triângulo ABC, tais

que Escolhendo dois quaisquer destes pontos; digamos, D e E (o que não perde em generalidade),

traçamos a reta DE. Esta interceptará a reta AB em um ponto F’. Nestas condições, decorre da primeira parte do

teorema de menelaus que: Mas, como devemos ter também , segue-se que:

Subtraindo 1 de ambos os membros desta igualdade obtemos Daí F’ = F, o que implica

dizer que os pontos D, E e F são colineares.

III. EXEMPLOS1. Uma Aplicação interessante do Menelaus é a dedução da condição ne-

cessária do teorema (T1) de CEVA. Mostramos ao lado esta deduçãopara um triângulo ABC de cevianas AD, BE e CF concorrentes em umponto P, conforme figura:

Demonstração:Utilizando o teorema de Menelaus nos triângulos ABD e ADC “cortados”respectivamente pelas transversais (ou secantes) CPF e BPE tem-se:

Multiplicando membro a membro e simplificando convenientemente, encontramos

2. Prove que: As retas suporte das alturas de um triângulo ABC são concorrentes.Este ponto de concorrência é denominado de Ortocentro

Prova:Sejam AD, BE e CF as retas suporte das alturas relativas aos lados BC, CA e AB respectivamente, onde D, E e Fsão os pés das alturas. Nestas condições, os triângulos retângulos BCE e ACD, com um ângulo agudo de vérticeC congruente, são semelhantes. Daí, CE/DC = BE/AD. Por outro lado, raciocinando de modo similar para os pares de triângulos retângulos: (∆CAF, ∆BAE) e (∆ABD, ∆CBF)semelhantes, obtemos AF/EA = CF/BE e BD/FB = AD/CF respectivamente.Decorre destas relações:

AF/FB ⋅ BD/DC ⋅ CE/EA = CE/DC ⋅ AF/EA ⋅ BD/FB = BE/AD ⋅ CF/BE ⋅ AD/CF = 1.Portanto, pelo recíproco do Teorema de CEVA no triângulo ABC, conclui-se que as retas suporte das alturas sãoconcorrentes.

3. Prove o Teorema de Van Obel: Se AD, BE, CF são cevianas de um triângulo ABC concorrentes em ponto S,

então AS

SD

AE

EC

AF

FB= +

A

B

C

DE

F

A

B CD

EF

AF

FB

BD

DC

CE

EA⋅ ⋅ = 1

AF

FB

BC

CD

DP

PAe

AP

PD

DB

BC

CE

EA⋅ ⋅ ⋅ ⋅= =1 1

AB

AF

AB

AF’.=AF

F B

AF

FB

’.=

AF

FB

BD

DC

CE

EA⋅ ⋅ = 1,

AF

F B

BD

DC

CE

EA

'.⋅ ⋅ = 1

AF

FB

BD

DC

CE

EA⋅ ⋅ = 1.

A

B CD

EF

P

Page 15: obmnivel3b

SISTEMA ANGLO DE ENSINO 15 ANGLO VESTIBULARES

Demonstração:Considere a propriedade P: “Se dois triângulos tem uma altura em comum, então suas áreas são proporcionais aoscomprimentos das suas bases correspondentes.” Da propriedade P e da figura auxiliar ao lado, tem-se:

[1]

mas [2]

e como, [3]

Portanto, de [1], [2], [3]:

4. (Olimpíada Colombiana) AD, BE e CF são cevianas concorrentes dotriângulo ABC.

Sabendo que calcule a razão de áreas:

Resolução

De tem-se

Assim, com estes resultados e o teorema de CEVA, temos:

Daí,

Usando agora a propriedade: “áreas de triângulos de alturas de comprimentos iguais são proporcionais aos com-primentos das correspondentes bases destes triângulos”, e as razões:

, podemos escrever sucessivamente:

[1]

[2]

[3]

Por outro lado, (DEF) + (AFE) + (BDF) + (CED) = (ABC) (ver figura)

Ou ainda

Portanto, da igualdade acima e [1], [2] e [3] obtém-se: ( )( )

– – .DEFABC

= + +

= =1 1

6121

914

1 67

17

( )( )

( )( )

( )( )

( )( )

DEFABC

ADEABC

BEDABC

CFEABC

+ + + =1

( )( )

( )( )

( )( )

CEDCAD

CADABC

CE

CA

DC

BC

CEDABC

⋅ = ⋅ ∴ = ⋅ =34

67

914

( )( )

( )( )

( )( )

BDFBCF

BCFABC

BD

BC

BF

AB

BDFABC

⋅ = ⋅ ∴ = ⋅ =17

13

121

( )( )

( )( )

( )( )

AFEABE

ABEABC

AF

AB

AE

AC

AFEABC

⋅ = ⋅ ∴ = ⋅ =23

14

16

AF

AB

BF

AB

CE

CA

AE

AC

BD

BCe

DC

BC= = = = = =2

313

34

14

17

67

, , , ,

BD

BCe

DC

BC= =1

767

AF

FB

BD

DC

CE

EA

BD

DC

BD

DC⋅ ⋅ ∴ ⋅ ⋅ ∴= = =1

21

31

116

.

AF

FB

BF

AB

CE

EAe

AE

AC= = = =2

113

31

14

, ,AF

ABe

CE

CA= =2

334

,

( )( )

.DEFABC

AF

ABe

CE

CA= =2

334

,

AS

SD

AE

EC

AF

FB= +

AF

FB

ASCSBC

= ( )( )

AE

EC

ABEEBC

ASEESC

ABE ASEEBC ESC

ABSSBC

= = = =( )( )

( )( )

( ) – ( )( ) – ( )

( )( )

,

AS

SD

ABSSBD

ASCSDC

ABS ASCSBD SDC

ABSSBC

ASCSBC

= = = ++

= + ⋅( )( )

( )( )

( ) ( )( ) ( )

( )( )

( )( )

A

B CD

EF

S

A

B CD

F

E

O

Page 16: obmnivel3b

SISTEMA ANGLO DE ENSINO 16 ANGLO VESTIBULARES

➲ atividades de aula1. a)

AD, BE CF são cevianas concorrentes, tais que: AF—

= 3, FB—

= 2, CE—

= 2 e EA—

= 4. Use o teorema de Ceva, para

calcular: a razão e a razão de áreas:

b)

D, F e E são pontos colineares, tais que: AF—

= 3, FB—

= 2, CE—

= 4 e EA—

= 2. Use o teorema de Menelaus, para

calcular: a razão e a razão de áreas:

2. a)

Os lados de um triângulo ABC são divididos em “partes iguais” conforme mostra a figura acima. Nestascondições, usando o teorema recíproco de CEVA, determine em qual das ternas de cevianas:

(AD, BE, CF) e AD’, BE’, CF’)encontramos concorrência das cevianas.

b)

Os lados de um triângulo ABC são divididos em “partes iguais”, F e D’ pontos marcados sobre os prolonga-mentos de BA e BC, conforme mostra a figura. Nestas condições, usando o teorema recíproco de Mene-laus, determine em qual das ternas de pontos:

(D, E, F) e ( D’, E’, F’)Encontramos colinearidade dos pontos.

A

C BD

E

E’

F’

D’

F

A

B CDD’

F’

F

E’

E

( )( )DFBDFC

BD

DC

A

F

E

DB C

3

2

2

4

( )( )ABDABC

BD

DC

A

4

E

2

CDB

2

F

3

0

Page 17: obmnivel3b

SISTEMA ANGLO DE ENSINO 17 ANGLO VESTIBULARES

3. (Treinamento Olímpico Americano) Usando o teorema de Ceva prove que as bissetrizes internas de um triângulosão concorrentes. (O ponto de concorrência é denominado de incentro).

4. (Treinamento Olímpico Americano) Seja M (distinto dos vértices) um ponto arbitrário sobre o lado AB de umtriângulo ABC. Se P e Q são pontos de intersecção das bissetrizes do ângulo BMC e AMC com lados BC e AC,respectivamente. Prove que as cevianas AP, BQ e CM do triângulo ABC são concorrentes.

5. (OBM) No triângulo ABC, M é o ponto médio do lado AC, D é um ponto sobre o lado BC tal que AD é bissetriz doângulo BÂC e P é o ponto de interseção de AD e BM. Sabendo que a área de ABC é 100, AB

—= 10, AC

—= 30, cal-

cule a área do triângulo APB.

6. (Olimpíada de Leningrado) A cevianas CH e BK respectivamente altura e mediana são traçadas em um triânguloacutângulo ABC. Sabe-se que BK

—–= CH

—–e ∠ KBC = ∠ HCB. Prove que o triângulo ABC é eqüilátero.

➲ atividades de casa1. a) Utilizando-se conveniente um dos teoremas (Ceva, Menelaus), calcule x e y nas figuras:

CF é mediana e BE é bissetriz F é ponto médio de ABAB— = 6, BC—– = 8 e CA— = 7 AB— = 6, BC—– = 8 e CA— = 7

2. (Olimpíada Regional Portuguesa) a) Sejam D e E pontos respectivamente sobre os lados BC e CA do triângulo ABC. Seja Z a interseção das

cevianas AD e BE (conforme figura abaixo).

Dado que determine as razões:

b) Seja ABC um triângulo retângulo em A. Sejam M, N e L pontos sobre os lados CA, AB e BC, respectivamente,tais que AN

—–= 2, NB

—–= 1, AC

—–= 4 e BL

—= x . Sabendo que as cevianas AL, CN, BM são concorrentes em um

ponto O e que BM é uma mediana, calcule x.

3. (Treinamento Olímpico Americano)a) Usando o teorema de Ceva prove que as medianas de um triângulo são concorrentes. (O ponto de concorrência

é denominado de baricentro).b) Sejam AD, BE e CF cevianas concorrentes de um triângulo ABC.

Sejam L, M e N pontos médios dos lados BC, CA e AB respectivamente.Sejam D’ simétrico de D em relação a L, E’ simétrico de E em relação a M e F’ simétrico de F em relação a N.Prove que as cevianas AD’, BE’ e CF’ são também concorrentes.

4. (Olimpíada Internacional de Matemática) As diagonais AC e CE do hexágono regular ABCDEF são divididasinternamente pelos pontos M e N respectivamente, tal que:

Determine λ, se B, M e N são colinearesAMAC

CNCE

= = λ .

AZ

ZDe

EZ

ZB

BD

DC

CE

EA= = 1

2,

A

B D C

E

Z

E

F

B

A

CD

y6

8 2

A

B CDx

F

E

P6

8

Page 18: obmnivel3b

SISTEMA ANGLO DE ENSINO 18 ANGLO VESTIBULARES

5. (Olimpíada americana-AIMEI) ABC é um triângulo com AB = 13, BC = 15, CA = 17. Pontos, D, E, F sobre os lados

AB, BC e CA respectivamente, tais que: Sabendo que

calcule a razão de áreas

6. a) (OBM) No triângulo ABC, a mediana e a altura relativas ao vértice A dividem o ângulo BÂC em três ângulos demesma medida. Determine as medidas dos ângulos do triângulo ABC.

b) (OBM) O triângulo ABC é retângulo em A. Dentre os pontos P pertencentes ao perímetro do triângulo, encontreaquele que minimiza a soma AP + BP + CP.

( )( )

.DEFABC

α β α βγ γ+ + = + + =23

25

2 2 2e ,ADAB

BEBC

eCFCA

= = =α β γ, , .

Aulas 5 e 6NÍVEL 3, SEGUNDA FASE, GEOMETRIA

I. INCÍRCULO — CIRCUNCÍRCULO — EXCÍRCULO 1. Definições

D1. Incírculo de um triângulo ABC é o círculo cuja circunferência tangência os 3 lados deste triângulo. Esta cir-cunferência é denominada de inscrita a este triângulo; o centro e o raio desta circunferência são denominadosrespectivamente de incentro e inraio do triângulo ABC.Na figura 1, I é o incentro e r é o inraio do triângulo ABC.

D2. Circuncírculo de um triângulo ABC é o círculo cuja circunferência “passa” pelos 3 vértices deste triângulo.Esta circunferência é denominada de circunscrita a este triângulo; o centro e o raio desta circunferência sãodenominados respectivamente de circuncentro e circunraio do triângulo ABC. Na figura 2, O é o cir-cuncentro e R é o circunraio.

(Figura 2)

A

R

R O R

B C

A

B

C

RO

RR

(Figura 1)

B D C

FE

A

I

rr

r

Page 19: obmnivel3b

SISTEMA ANGLO DE ENSINO 19 ANGLO VESTIBULARES

D3. Excírculo de um triângulo ABC é o círculo cujacircunferência é exterior ao triângulo ABC, mastangente a um lado e aos prolongamentos dos ou-tros dois lados.

Esta circunferência é denominada de exinscrita aotriângulo ABC.

O centro e o raio desta circunferência são respec-tivamente denominados de excentro e exraiodo triângulo ABC.

Na figura 3, temos:

Ia, Ib e Ic são os centros das exincritas ao triân-gulo ABC tangentes respectivamente aos lados a,b e c.

ra, rb e rc são os raios das exincritas ao triânguloABC tangentes respectivamente aos lados a, b e c.

I é o centro do incírculo e r o seu inraio.

2. Propriedades

P1. Os segmentos tangentes a uma circunferência conduzidos por pontoqualquer externo são congruentes. (figura 4)

P2. O centro do incírculo do triângulo ABC é a interseção das bisse-trizes internas.

P3. O centro do circuncírculo do triângulo ABC é a interseção dasmediatrizes dos lados deste triângulo.

P4. O centro de um excírculo do triângulo ABC é a interseção de duasbissetrizes externa e uma interna.

P5. Seja ABC um triângulo com incírculo tangenciando os lados BC, CA eAB em X, Y e Z respectivamente. Se este triângulo tem lados de com-primentos a, b e c, com a = BC

—–, b = AC

—, c = AB

—, e semiperímetro p

entãoAY—

= AZ—

= p – a

BZ—–

= BX—–

= p – b (figura 5)

CX—–

= CY—

= p – c

P6. Seja ABC um triângulo com um excírculo tan-genciando o lado CA em Yb, o prolongamento deBC em Xb e o prolongamento de BA em Zb.

Se este triângulo tem lados de comprimentos a, be c com a = BC

—–, b = AC

—, c = AB

—, e semiperímetro

p, então

CYb—–

= CXb—–

= p – a

AYb—–

= AZb—–

= p – c (figura 6)

BZb—–

= BXb—–

= p

Icra

A

B

I

rb

Ib

rc

r

C

E

Ia

(Figura 3)

A

0

B

P

(Figura 4)

A

Z

Y

XB C(Figura 5)

Zb

Xb

Yb

A

B C

(Figura 6)

Page 20: obmnivel3b

SISTEMA ANGLO DE ENSINO 20 ANGLO VESTIBULARES

II. PROPRIEDADES DE TANGÊNCIA 1. Tangente a uma Circunferência λ• T ∈ λ e OT ⊥ PT

• A reta PT é tangente a λ.• T ponto de tangência.

• PO—–2 = PT

—2 + R2 (R raio de λ)

• PA— ⋅ PB

—= PT—2 (Potência de P em relação a λ)

2. Circunferências (λ1 e λ2) Secantes• O e R centro e raio respectivamente de λ1.• O’ e R’ centro e raio respectivamente de λ2.• PC tangente a λ1 em C por um ponto P externo. (OC ⊥ PC e C ∈ λ 1).• PD tangente a λ2 em D por um ponto P externo. (O’D ⊥ PD e D ∈ λ 2).

• PC—–2 = PD

—–2 = PM—– ⋅ PN

—–(Potência de P em relação a λ1 e λ2)

• OT é mediatriz do segmento MN (OM—– = ON—– = R e O’M—– = O’N—— = r)• Existem duas tangentes comuns a λ1 e λ2:

Uma delas e a reta AB (A ∈ λ 1, B ∈ λ 2, OA ⊥ AB, O’B ⊥ AB) e a outra a simétrica de AB em relação a reta O O’.

3. Tangência Externa

3 tangentes comuns: AB, CD e EFEA— = ET— = EB— (propriedade)

4. Tangência Interna

1 tangente comum: EF

E

T

F

O O’’

(λ2)

(λ)

(λ2)

(λ1)A

E B

P

DF

C

O TO’’ O’

(λ1)

A

(λ2)

LB

N

T

r

O’

DM

O

C P

r

R

R

T

O A

P

B

(λ)

Page 21: obmnivel3b

SISTEMA ANGLO DE ENSINO 21 ANGLO VESTIBULARES

5. Tangência Externa e Interna

λ1 e λ2 são tangentes externamente.λ2 e λ são tangentes internamente.λ1 e λ são tangentes internamente.(O) centro de λ2, (O”) centro de λ e (O’) centro deλ1, são pontos colineares.

6. Tangentes Comuns a DuasCircunferências Externas

Existem quatro tangentes comuns,duas são diretas (AB e CD)duas são transversais (EF e GH).• A, B, C e D são pontos de tangência.• S é a intersecção das retas tangentes (diretas)

AB e CD as circunferências de centros O e P.• T é a intersecção das retas tangentes (diretas)

AB e CD as circunferências de centros O e P.

A

EH

B

X O Y S U PT

G

C

DF

V

(λ2)

(λ1)A

E B

P

DF

C

O TO’’O’

(λ)

III. PONTOS NOTÁVEIS DE UM TRIÂNGULO

D5. O triângulo cujos os vértices são os pontos médios dos lados do triângulo ABC é denominado de ‘triângulomedio’. Enquanto que o triângulo cujos os vértices são os pés das alturas do triângulo ABC é denominado detriângulo ‘órtico’.

2. Posição dos Pontos Notáveis em Relação ao Triângulo ABC — Ortocentro (H) de um triângulo ABC

AcutânguloObtusângulo Retângulo

H é externo ao ∆ABC H é interno ao ∆ABC H é o vértice do ângulo reto

H

E F

A

B CD

A

F

E

B D C

H

H = B C

E

A

1. DefiniçõesD1. Baricentro: Ponto de intersecção das media-

nas de um triângulo

D2. Ortocentro: Ponto de intersecção das retas su-portes das alturas de um triângulo

D3. Incentro: Ponto de intersecção das bissetrizesinternas de um triângulo

D4. Circuncentro: Ponto de encontro das mediatrizesde um triângulo.

Propriedades

As medianas de um triângulo tem um único ponto emcomum. Se G é o baricentro de um triângulo ABC e M é pontomédio do lado BC então AG = 2 GM.

As retas suportes das alturas de um triângulo tem umúnico ponto em comum. O simétrico do ortocentro em relação a qualquer ladopertence a circunferência circunscrita a este triângulo

As bissetrizes internas de um triângulo tem um únicoponto comum.O incentro eqüidista dos lados de um triângulo

As mediatrizes dos lados de um triângulo tem um únicoponto em comum.O circuncentro de um triângulo ABC eqüidista dos vér-tices deste triângulo

Page 22: obmnivel3b

SISTEMA ANGLO DE ENSINO 22 ANGLO VESTIBULARES

— Circuncentro (O) de um triângulo ABC

— O incírculo I (incentro) e o baricentro (G) são sempre pontos internos ao triângulo ABC.

IV. LEITURA COMPLEMENTARVeremos abaixo propriedades não usualmente mencionados nas aulas do nosso ensino médio. Seus resultados

(“pérolas”) são extremamente importante para um estudo mais profundo da Geometria euclideana contemporânea.Além disso, o seu conhecimento tem sido fundamental na resolução de vários problemas que aparecem na fase final(que não é o nosso objetivo aqui) das olimpíadas de matemática.

1. Triângulo MédioSejam D, E, F pontos médios dos lados BC,CA e AB de um triângulo ABC.Propriedade: O triângulo DEF, denominado de triângulo médio, é semelhante ao triângulo ABC. A razão desta

semelhança é 1 : 2, com

2. Simétricos do OrtocentroSejam Ha, Hb e Hc os simétricos do ortocentro H de um triângulo ABC em relação aos lados de comprimento

a = BC—–

, b = CA—

e c = AB—

.Propriedade: Ha, Hb e Hc pertencem à circunferência circunscrita ao triângulo ABC.

3. Reta de EULEROrtocentro (H), Baricentro (G) e Circuncentro (O) de qualquer triân-

gulo ABC são colineares, isto é, pertencem a uma mesma reta. A retaque contém estes pontos é denominada de reta de EULER

Além disso, o baricentro divide a distância do ortocentro ao cir-cuncentro na razão 2 : 1. Na Figura 11, em relação aos lados BC e CA te-mos respectivamente:• As retas mediatrizes OM e ON. • As alturas AD e BE.• As medianas AM e BN.• H (ortocentro), G (baricentro) e O (circuncentro). • A reta de EULER passando por H, G e O.

4. Circunferência dos nove pontosOs nove pontos: pés das 3 três alturas de um triângulo ABC; (D, E, F)pontos médios dos 3 lados (A’, B’, C’) epontos médios dos segmentos HA, HB e HC , (L, M, N)

onde H é ortocentro. Pertencem a uma mesma circunferência deno-minada de “circunferência dos nove pontos”

O centro desta circunferência pertence a reta de EULER e o seuraio é a metade do circunraio do triângulo ABC.

DEAB

EFBC

FDAC

= = = 12

.

obtusânguloO é externo ao ∆ABC

AO = OB = OC = R(R é o circunraio)

A

B RO

RR

CA

B C

R

R

RO

B

AR O R

R

C

acutânguloO é interno ao ∆ABC

OA = OB = OC = R(O é o circuncentro)

retânguloO é o ponto médio de AC

BO é a mediana relativa a ACAC = 2.BO— —

A

H G

O

N

E

B D M C

(FIGURA 11)

A

C’ KB’

N E

HMLF

B D A’ C

Page 23: obmnivel3b

SISTEMA ANGLO DE ENSINO 23 ANGLO VESTIBULARES

5. Reta de SimsonSeja ABC um triângulo. P um ponto do seu circuncírculo. D, E e F são as projeções ortogonais de P sobre as retas

suporte dos lados BC, CA e AB respectivamente. Propriedade: Os pontos D, E e F estão sobre uma reta. Esta reta é denominada de “Simson”.

6. Teorema de EulerA distância do incentro ao circuncentro de um triângulo

ABC é

R e r são respectivamente circunraio e inraio do triângulo ABC.

(Leonardo. Euler matemático alemão (1707-1783))

V. EXEMPLOS1. (Olimpíada Colombiana) Seja ABC um triângulo retângulo, com ∠ ABC = 90º. Seja H pé da altura relativa ao lado AC.

Seja D ponto de interseção de uma paralela a AB por C com a reta BH . Seja E ponto de interseção de umaparalela a BC por D com a reta AC. Demonstrar que as retas AD e BE são perpendiculares.

Resolução

Seja F a interseção da reta BE com a reta AD.Seja G a interseção da reta DE com a reta AB.Desde que DE é perpendicular a AB, pois é paralela a BC, e AH éperpendicular a BD, pois BH é a altura de B relativa ao lado AC, então,DG e AH são alturas do triângulo ABD. Nestas condições, conclui-seque E é o ortocentro de ABD e portanto BF a outra altura deste triân-gulo. Consequentemente a reta BE suporte desta altura é perpendicu-lar a AD, o que finaliza a demonstração.

2. (Treinamento para a OBM) A bissetriz do ângulo A do triângulo ABC intercepta o lado BC no ponto U. Prove que amediatriz do segmento AU, a perpendicular a BC por U e o diâmetro do círculo circunscrito que contem A sãoconcorrentes

Resolução

I. Suponhamos inicialmente que AB—

≠ AC—

.Seja λ circuncírculo de ABC.Seja R o circunraio de ABC.Seja O circuncentro de λ.Seja N interseção do prolongamento de AU com λ.Seja LU a reta perpendicular a BC por U.Seja S a interseção da reta OA com a reta LU.Nestas condições, como N pertence a bissetriz do ângulo A,então NB

—–= NC

—–. Assim, N também pertence a mediatriz de BC.

Daí, ON ⊥ BC. Portanto, ON//LU, conseqüentemente o triânguloOAN é semelhante ao triângulo SAU.

Por outro lado, ON—–

= OA—

= R, logo o triângulo OAN é isósceles de base NA, conseqüentemente o triângulo SAUsemelhante a OAN, também é isósceles, com SA—– = SU—–, o que implica dizer que, S pertence a mediatriz de AU,mas como já pertence a interseção de LU com a reta OA, conclui-se que S é o ponto de concorrência das retasOA, LU e a mediatriz de AU.

II. No caso, de AB—

≠ AC—

, a reta AU é perpendicular a BC por U e U é ponto médio de BC. Logo, consequente-mente as retas OA, LU e AU são coincidentes. Portanto, nestas condições, é imediato que as retas: OA, LU e amediatriz de AU são concorrentes e tem o ponto médio do segmento AU como ponto de concorrência.Portanto, com as conclusões de (I) e (II), chegamos ao final da demonstração.

R R r( – ).2

B

RAI

r

R

C

O

AF

G E D

H

B C

A

L

S

O

B CU

Page 24: obmnivel3b

SISTEMA ANGLO DE ENSINO 24 ANGLO VESTIBULARES

➲ atividades de aula1. a) Seja ABC um triângulo com incírculo tangenciando os

lados BC, CA e AB em X, Y e Z respectivamente

Mostre que:

a = BC—–

, b = AC—

, c = AB—

, p é o semiperímetro

b) (Ponto de Gergonne)Sejam X, Y, Z pontos de tangência do incírculo do triân-gulo ABC com seus os lados Prove que as cevianas AX, BYe CZ são concorrentes, isto é, tem um ponto S em co-mum.

Nota: Este ponto é denominado de ponto de Gergonne,em homenagem ao matemático Joseph Dias Gergonne(1771-1859), enquanto que as cevianas AX, BY e CZ sãodenominadas de cevianas de Gergonne.

2. a) Seja ABC um triângulo com um excírculo tangencian-do o lado CA em Yb, o prolongamento de BC em Xb e oprolongamento de BA em Zb.

Mostre que:

Com a = BC—–

, b = AC—

, c = AB—

, e o semiperímetro p.

b) (Ponto de Nagel)

Sejam Xa, Yb e Zc pontos de tangência dosexcírculos do triângulo ABC com os lados BC,CA e AB. Prove que as cevianas AXa, BYb eCZc são concorrentes, isto é, tem um ponto Nem comum.

Nota: Este ponto é denominado de ponto deNagel enquanto que as cevianas AXa, BYb eCZc são denominadas de cevianas de Nagel.(1803-1882)

CY CX p a

AY AZ p c

BZ BX p

b b

b b

b b

= =

= =

= =

AY AZ p a

BZ BX p b

CX CY p c

= =

= =

= =

A

Z

Y

B X C

O

Z

O

Y

B X C

A

S

Zb

A Ib

rb

XbC

B

Yb

Icrc

Zc

A

Zc

YbN

Xa

ra

Ia

Xb

rb

Ib

C

B

Page 25: obmnivel3b

SISTEMA ANGLO DE ENSINO 25 ANGLO VESTIBULARES

3. (Olimpíada Russa) Seja ABC um triângulo e D um ponto de BC. Demonstrar que: Se os incírculos dos triângulosABD e ACD são tangentes entre si, então D é o ponto de tangência do incírculo do triângulo ABC com o lado BC.

4. a) Se G é o baricentro de um triângulo ABC, então os ∆GAB, ∆GBC e ∆GCA tem áreas iguais.b) (Olimpíada do Reino Unido) Dois círculos secantes C1 e C2 tem uma tangente em comum, que tangencia C1

em P e C2 em Q. N e M são os pontos de interseção destes círculos, sendo N mais próximo de PQ do que M.Prove que os triângulos MNP e MNQ tem áreas iguais.

5. (Olimpíada Peruana) Em um triângulo ABC o incentro e o circuncentro são simétricos em relação ao lado AB.Achar, em graus, a medida do maior ângulo do triângulo ABC.

6. a) (Olimpíada regional Portuguesa) ABC é um triângulo acutângulo de ortocentro H e circuncentro O. Prove que osimétrico de H em relação a AB pertence ao circuncírculo de ABC.

b) (Olimpíada Colombiana) Na figura abaixo, ABCD é um quadrado. Sendo BE = BF, determine a medida doângulo ∠ AGE, em graus.

➲ atividades de casa1. a) ABC é um triângulo retângulo em B. Dado que BC = 4 e AB = 3, determine o raio do excírculo tangente ao lado BC.

b) (Olimpíada Espanhola) Em um triângulo retângulo, c é o comprimento da hipotenusa, a e b são os compri-mentos dos catetos e d é o comprimento do diâmetro do incírculo. Prove que: a + b = c + d.

c) (Olimpíada Regional Italiana) ABC é um triângulo, com BC—–

� AB—

.Sejam Y e Yb pontos de tangência respectivamente do Incírculo e de um excírculo do triângulo ABC com o lado AC.Calcule o comprimento do segmento YYb em função dos lados de comprimentos a, b e c com a = BC

—–, b = AC

—–e

c = AB—

.

2. a) (Sangaku-problema Japonês) Três círculos de raios a, b e c, com c � b � a, são mutuamente tangentes entre si ea uma reta.

Prove que: .

b) (OBM) Seja ABCD um quadrado. Escolhemos pontos M, N, P, Q respectivamente sobre AB, BC, CD e DA, demodo que as circunferências circunscritas aos triângulos MBN e PDQ sejam tangentes exteriormente. Mostreque MN + PQ � AC.

3. (Olimpíada Russa) Seja ABC um triângulo e D um ponto de BC. Prove que:Se D é o ponto de tangência do incírculo do triângulo ABC com o lado BC, então os incírculos dos triângulos ABDe ACD são tangentes entre si.

4. (Problemas Olímpicos franceses) Considere um triângulo acutângulo ABC. Trace as alturas AA’ e BB’, seja H oortocentro do mesmo. Seja M o ponto médio do lado AB e N o ponto médio do segmento CH. Prove que MN éperpendicular a A’B’.

1 1 1

c a b= +

A B E

D C

GF

a

c

b

Page 26: obmnivel3b

SISTEMA ANGLO DE ENSINO 26 ANGLO VESTIBULARES

5. (Olimpíada do Reino Unido)a) Considere um triângulo ABC com o ângulo de vértice A medindo 120º. As bissetrizes dos ângulos internos A, B

e C intersectam os lados opostos em D, E e F, respectivamente. Prove que o círculo de diâmetro EF passa por D.b) (Olimpíada Cone Sul) Seja C uma circunferência de centro O, AB um diâmetro dela e R um ponto qualquer em

C distinto de A e de B. Seja P a interseção da perpendicular traçada por O a AR. Sobre a reta OP se marca oponto Q, de maneira que QP é a metade de PO e Q não pertence ao segmento OP. Por Q traçamos a paralela aAB que corta a reta AR em T.Chamamos de H o ponto de interseção das retas AQ e OT.Provar que H, R e B são colineares.

6. (Olimpíada Espanhola) Pelo baricentro G de um triângulo ABC traça-se uma reta que corta o lado AB em P e o

lado AC em Q. Demonstre que PB

PA

QC

QA⋅ � 1

4.

Aulas 7 e 8NÍVEL 3, SEGUNDA FASE, GEOMETRIA

I. QUADRILÁTEROS1. Elementos e Definição

Quadrilátero é um polígono com quatro lados. Na figura ao lado:A, B, C e D, ........... são vértices consecutivos.ABC, BCD, DÂE, ........ são ângulos internos consecutivos.AB, BC, CD e DA, ........ são lados consecutivos.AC e BD, ................ são diagonais

2. Classificação dos Quadriláteros

TrapézioÉ o quadrilátero que tem dois lados paralelos. Na figura abaixo, AB é paralelo a CD, AB (base maior) e DC (base

menor).

Os trapézios recebem nomes de acordo com os triângulos que têm características semelhantes.

Um trapézio pode ser:

• Retângulo: dois ângulos retos• Isósceles: lados não paralelos congruentes• Escaleno: lados não paralelos diferentes

TrapézioRetângulo

TrapézioIsósceles

TrapézioEscaleno

D C

A BTrapézio

A

BC

D

Page 27: obmnivel3b

SISTEMA ANGLO DE ENSINO 27 ANGLO VESTIBULARES

ParalelogramoÉ o quadrilátero que tem lados opostos paralelos. Num paralelo-

gramo, os ângulos opostos são congruentes. Os paralelogramos maisimportantes recebem nomes especiais:

• Losango: Paralelogramo com quatro lados congruentes

• Retângulo: Paralelogramo com quatro ângulos retos.

• Quadrado: Paralelogramo com quatro lados congruentes e quatro ângulos retos

3. Propriedades

P1. Ao unir os vértices opostos de um quadrilátero qualquer, obtemossempre dois triângulos e como a soma das medidas dos ângulosinternos de um triângulo é 180º, concluímos que a soma dos ângulosinternos de um quadrilátero é igual a 360º

P2. Retângulo é um paralelogramo de diagonais congruentes.

P3. Losango é um paralelogramo de diagonais perpendiculares.

P4. Os pontos médios dos lados sucessivos de um quadriláteroconvexo são vértices de um paralelogramo. Este paralelogramoé denominado de paralelogramo de VARIGNON (Pierre Varignonmatemático Francês (1654 -1722))

II. PONTOS CONCICLICOS1. Definições:

D1. Três ou mais pontos de um plano são conciclicos se, e somente se, existe uma circunferência que “passa”por eles.

D2. Dizemos que um quadrilátero é cíclico se, e somente se, seus vértices são conciclicos. Um quadrilátero ABCDcíclico pode ser convexo ou côncavo (cruzado)

D3. Chamamos de Circuncírculo de um quadrilátero cíclico a circunferência que “passa” por seus vértices. O centro eo raio desta circunferência são denominados respectivamente de circuncentro e circunraio do quadrilátero.

β

C

B

D

α + β = 180°

C

B

A

D

α

αβ

β

ABCD cíclico convexo ABCD cíclico cruzado

A C

D

B

Losango Retângulo

D C

A B

C

B

D

A

Quadrado

D C

A B

Paralelogramo

A

B

C

D

G

F

E

D DGFE paralelogramo

D, E, F, G pontos médiosdos lados de ABCD

Page 28: obmnivel3b

SISTEMA ANGLO DE ENSINO 28 ANGLO VESTIBULARES

III. QUADRILÁTERO CIRCUNSCRITÍVELDefinições

Um quadrilátero é circunscritível a uma circunferência λ se, e somente se,cada um de seus lados é tangente a λ.

Esta circunferência é denominado de incírculo do quadrilátero.O centro e o raio de λ são denominados respectivamente de incentro e inraio

deste quadrilátero. Na figura ao lado, a circunferência λ é o incírculo do quadriláteroABCD.; O é seu incentro e r seu inraio.

IV. PROPRIEDADESP1. Quatro pontos A,B, C e D (distintos dois a dois) são conciclicos se, e somente se, ocorrer uma das situações abaixo:

• ACB = ADB, se C e D pertencem ao mesmo • ACB + ADB = 180º, se C e D pertencem asemiplano determinado pela reta AB semiplanos opostos determinados por AB

P2. ABCD é um quadrilátero cíclico convexo se, e somente se,∠ A + ∠ C = ∠ B + ∠ D = 180º

P3. Se ABCD é um quadrilátero cíclico cruzado, então∠ A = ∠ C e ∠ B = ∠ D.

P4. Se ABCD é um quadrilátero convexo cíclico e P ponto deinterseção dos prolongamentos de BC e AD, conforme afigura ao lado, então os triângulos: PAB e PCD são seme-lhantes.

P5. (Teorema de Ptolomeu) Para qualquer quadriláteroABCD convexo o produto de duas diagonais é menor ouigual a soma dos produtos dos lados opostos. A igualdade vale se, e somente se, ABCD é cíclico, isto é,Se ABCD é um quadrilátero cíclico convexo, então

AC— ⋅ BD— = AB— ⋅ CD— + BC— ⋅ AD—

reciprocamente, se os lados e diagonais de um quadrilá-tero ABCD são tais que AC— ⋅ BD— = AB— ⋅ CD— + BC— ⋅ AD—, entãoABCD é um quadrilátero cíclico convexo

DemonstraçãoNa figura ao lado, traçamos a reta AE, de modo que ∠ DAE = ∠ CAB,interceptando o segmento DB em E. Então, nestas condições: temosas semelhanças:• dos triângulos DAE e CAB, donde obtém-se:

AD— ⋅ BC— = ED— ⋅ AC— [1]• dos triângulos ADC e AEB, donde obtém-se:

AB— ⋅ CD— = BE— ⋅ AC— [2]Como BE— + ED— = BD—, resulta da adição membro a membro de [1] e [2]:

AC— ⋅ BD— = AB— ⋅ CD— + BC— ⋅ AD—

A recíproca deste teorema também é verdadeira. Sugerimos ao leitor como exercício.

α

α

B

C

D

A

2β D

BA

C

α

β

A

(λ)

B

D

E

C

FG

D

r

r r

r

O

A

BC

D

P

A

B

D

C

p

s

q

r

A

B

D

C

a

d

b

cE

Page 29: obmnivel3b

SISTEMA ANGLO DE ENSINO 29 ANGLO VESTIBULARES

P6. (Outro teorema de Ptolomeu) Se os comprimentos dos lados de um quadrilátero cíclico, ABCD, convexo

são, a = AB—, b = BC—, c = CD— e d = DA— , (figura), então

P7. (Teorema de PITOT) Um quadrilátero ABCD é circunscritível a uma circunferência se, e somente se,AB— + CD— = AD— + BC—

Demonstração

I. (Condição Suficiente)

ABCD é Circunscritível ⇒ AB— + CD— = AD— + BC—

Sejam M, N, P e Q os pontos de tangência dos lados AB,BC, CD e DA, como na figura ao lado. Assim, da propriedade:“Segmentos tangentes a uma circunferência conduzidos porum ponto externo desta são congruentes”, resulta:

AB— + CD— = AM— + BM— + CP— + DP—

= AQ— + BN— + CN— + DQ—

= (AQ— + DQ—) + (BN— + CN—)= AD— + BC—

o que finaliza a demonstração da condição suficiente.

II. (Condição necessária):

AB— + CD— = AD— + BC— ⇒ ABCD é circunscritível

DemonstraçãoConsidere um quadrilátero ABCD com AB— + CD— = AD— + BC— e uma circunferência λ tangente aos lados AB, AD eCD (isto é sempre possível; seu centro é o ponto de interseção das bissetrizes de dois ângulos  e D) (ver figura)Temos duas possibilidades a considerar sobre BC:Primeira Possibilidade: BC é tangente a circunferência λ,Neste caso, é imediato que ABCD é circunscritível.Segunda possibilidade: BC não é tangente a circunferência λ, Neste caso,construímos uma tangente BE (E sobre a reta CD).Daí, ABCE é um quadrilátero circunscritível. Logo, pela hipóteseacima e a condição suficiente, já demonstrada, tem-se:AB + CD = AD + BC ( hipótese) e AB + DE = AD + BE (ABED é circunscritível)

Donde obtém-se sucessivamente CD – DE = BC – BE, ou ainda CD + BE = BC + DE (*)Se E pertence ao segmento CD; CD = CE + DE, então de (*) BC = EC + BE, o que é um absurdo.Se E não pertence ao segmento CD; DE = CE + CD, então de (*) BE = BC + EC, o que é um absurdo.O absurdo em ambos casos ocorre por que no triângulo BCE, não podemos ter um lado como soma dos outrosdois.(Desigualdade triangular). Portanto, como C esta sobre a reta DE, conclui-se que necessariamente devemoster C = E; consequentemente ABCD é circunscritível.As duas possibilidades estudadas acima finalizam esta demonstração.

P8. (Teorema de Miquel) Seja ABC um triângulo P, Q e R são três pontos situados sobre os lados BC, CA e ABrespectivamente. Então as circunferências circunscritas aos triângulos ARQ, BPR, CQP passam por um pontocomum.

ACBD

ad bcab cd

= ++

A

D

B

P

C

NM

Q

BA

DE C

A

BC

R

Q

P

T

Page 30: obmnivel3b

SISTEMA ANGLO DE ENSINO 30 ANGLO VESTIBULARES

P9. (Teorema de Brahmagupta) Se um quadrilátero convexocíclico tem diagonais perpendiculares que se interceptam em umponto T então a reta perpendicular a qualquer lado por T divide olado oposto ao meio.

(Brahmagupta matemático hindu — viveu no século VII).

P10. (Teorema de Bretscheneider) A área de um quadriláteroconvexo ABCD qualquer de lados a, b, c e d com semiperimetro p é

dada por:

CONSEQUÊNCIAS IMPORTANTESI) Dentre os quadriláteros de lados a, b, c e d o de maior área é o

quadrilátero cíclico convexo. A área de um quadrilátero cí-clico convexo de semiperimetro p é

(Este resultado é devido a Brahmagupta)II) A área de um quadrilátero, cíclico e circunscritível, de lados a, b, c

e d é igual a(Um quadrilátero, cíclico e circunscritível, é denominado de Qua-drilátero Bicentrico.)

V. EXEMPLOS1. Um quadrilátero tem lados medindo 1cm, 4cm ,7cm e 8cm. Qual é a área máxima deste quadrilátero?

ResoluçãoUm quadrilátero com lados de comprimentos dados por a,b,c e d tem área máxima, quando é convexo e cíclico.

Assim, a sua área é dada por

Portanto, fazendo a = 1, b = 4, c = 7 e d = 8 na expressão acima, resulta que a área máxima deste quadrilátero, emcm2, é 18.

2. (OBM) Sejam H, I e O o ortocentro, o incentro e o circuncentro do triângulo ABC, respectivamente. A reta CI cor-ta o circuncírculo de ABC no ponto L, distinto de C. Sabe-se que AB = IL e AH = OH. Determine os ângulos dotriângulo ABC.

Resolução OficialSendo α, β e γ as medidas dos ângulos internos nos vértices A, B e C,

respectivamente, temos

Logo BL—

= IL—

, e como BL—

= AL—

e IL—

= AB—

, concluímos que o triânguloABL é eqüilátero, logo o arco AB mede 60º e, portanto, ∠ ACB = 120º.O quadrilátero CXHY é cíclico, onde X e Y são os pés das alturas traça-das de A e B. Logo ∠ AHB = 180º – 120º = 60º.Como ∠ AOB = 120º, concluímos que o quadrilátero OAHB é cíclico. (Istotambém pode ser provado, por exemplo, utilizando-se a propriedade deque o simétrico de H em relação a AB pertence ao circuncírculo de ABC).Isto implica que ∠ AHO = ∠ ABO = 30º, e como OH— = AH—, temos:

∠ AOH = ∠ OAH = 75º.Finalmente, ∠ BAC = ∠ OAH – ∠ OAB – ∠ XAH

∠ BAC = 75º – 30º – 30º = 15º;e ∠ ABC = 180º – 120º –15º = 45º.

( ) ( ) .∠ ⋅ ∠ = +

IBL BILα β

2

(p – a) (p – b) (p – c) (p – d) onde p

12

(a b c d)⋅ ⋅ ⋅ = + + +

abcd.

( – ) ( – ) ( – ) ( – )p a p b p c p d⋅ ⋅ ⋅

( – ) ( – ) ( – ) ( – ) – cosˆ

p a p b p c p d abcd C⋅ ⋅ ⋅ +

2

2

T

A

D

E

B CF

TE perpendicular a ADF ponto médio de BC

A

D

BC

O

K

QuadriláteroBicentrico

L

A B

XY

I

H

C

O

Page 31: obmnivel3b

SISTEMA ANGLO DE ENSINO 31 ANGLO VESTIBULARES

3. (Olimpíada Russa) Um ponto M é escolhido no interior de um triângulo ABC de modo que:

e a reta AM contenha o circuncentro do triângulo BMC. Nestas condições, prove que M é o incentro do triângulo ABC.

ResoluçãoSeja O o circuncentro do triângulo MBC. ( vide figura ao lado).

Então,∠ BOC = ∠ BOM + ∠ MOC

= 180º – 2 ⋅ ∠ BMO + 180º – 2 ⋅ ∠ CMO= 360º – 2 ⋅ ∠ BMC= 180º – ∠ CAB

Daí conclui-se que o quadrilátero ABOC é cíclico.

Consequentemente, AM é bissetriz do ângulo BÂC, pois BO— = OC—; e que CM é bissetriz do ângulo ACB, pois∠ BCA = ∠ BOM = 2 ⋅ ∠ BCM.

Portanto, M é o incentro do triângulo ABC.

4. (Olimpíada Mexicana) Seja S uma circunferência e AB um diâmetro dela. Seja t a reta tangente a S em B econsidere dois pontos C, D em t tais que B está entre C e D. Sejam E e F as intersecções de S com AC e AD esejam G e H as intersecções de S com CF e DE. Demonstrar que AH = AG.

ResoluçãoComo ∠ AGB = ∠ AHB = 90º e os triângulos AGB e AHB tem o lado comum AB, será suficiente a demonstração daigualdade: ∠ ABH = ∠ ABGPara concluirmos que os triângulos AGB e AHB são congruentes e consequentemente que AH = AG.Como AEBH e AGBF são quadriláteros cíclicos, então ∠ AEH = ∠ ABH e ∠ GBA = ∠ GFA.Agora, como ∠ AEH + ∠ CED = 180º = ∠ GFA + ∠ CFD, se ∠ CED = ∠ CFD, então ∠ AEH = ∠ GFA.Para demonstrar que ∠ GED = ∠ CFD basta demonstrar que CEFD é cíclico. Isto decorre da semelhança entre ostriângulos ABE e ABC como também da semelhança entre os triângulos AFB e ABD.Então, da primeira semelhança, AB2 = AE ⋅ AC e da segunda semelhança, AB2 = AD ⋅ AF. Consequentemente,AE ⋅ AC = AD ⋅ AF e CEFD é cíclico.Nota: Outro modo de provar que CEFD é cíclico é observando que os triângulos AEB e ABC são semelhantes,pois são retângulos e o fato de ∠ EAB = ∠ CAB. Portanto ∠ ABE = ∠ BCA.Por outro lado ∠ ABE = ∠ AFE pois ABEF [e cíclico e ambos ângulos subtendem o mesmo arco. Logo

∠ ABE = 180º – ∠ AFE = 180º – ∠ ABE = 180º – ∠ ACB = 180º – ∠ ECD.Portanto, ∠ AEH + ∠ CED = 180º e CEFD é cíclico.

5. (INSTITUTO MILITAR DE ENGENHARIA-IME) Um quadrilátero convexo ABCD está inscrito em um circulo de diâ-metro d. Sabe-se que AB— = BC— = a, AD— = d e CD— = b, com a, b e d diferentes de zero. Demonstre que d2 = bd + 2a2

Sejam B´ e C´ os simétricos de B e C em relação ao centro O do cir-cuncirculo de ABCD.Estes pontos pertencem ao mesmo circuncírculo e juntamente comos demais vértices do quadrilátero formam um hexágono convexoABCDB´C´,em que:

AB = BC = DB´= B´C´= a e CD = C´AOs lados deste hexágono dividem a circunferência em 4 arcos demedida α e 2 arcos de medida β, conforme mostra a figura.

Nestas condições, 4α + 2β =360º, ou 2α + β =180º.Daí, segue-se que:• ABB´C´é um trapézio isósceles de bases b e d. e• BC´B´ é um triângulo retângulo em C´.Portanto, do teorema de Ptolomeu e Pitágoras, aplicados no trapézio e no triângulo, respectivamente, obtém-se

BC’— 2 = d2 – a2 = bd + aa

Donde resulta, d2 = bd + 2a2, finalizando a demonstração.

∠ ∠= +BMC

BAC902

º ( )

A

B

C

OM

A

B C

D

C’ B’a

b

a

a

b

a

α

β

α

α

β

α

O

d

Page 32: obmnivel3b

SISTEMA ANGLO DE ENSINO 32 ANGLO VESTIBULARES

➲ atividades de aula

1. (Olimpíada Regional Peruana) A, D, E e B são conciclicos,∠ AEB= 90º, ∠ ABD = 20º e ∠ DEC = x (graus), conforme afigura 1. Calcule x

2. (Olimpíada Australiana) Seja ABC um triângulo eqüiláteroinscrito numa circunferência. Seja M um ponto do menor arcode extremidades BC. Prove que AM

—= BM

—+ CM

—. (figura 2)

3. (OBM) Seja ABCD um trapézio retângulo de bases AB e CD, com ângulos retos em A e D. Dado que a diagonal

menor BD é perpendicular ao lado BC, determine o menor valor possível para a razão

4. (Treinamento Olímpico Brasileiro) Um quadrilátero ABCD é especial se for cíclico e tiver diagonais perpendicu-lares. Prove que:Se ABCD é especial, T é o ponto de interseção das diago-nais, P ponto médio de BC e O seu circuncentro, então:

a) a reta perpendicular ao lado AD por T contém o ponto P.

b)

Nota: A propriedade de quadriláteros que pede-se paraprovar na parte (a) é devida ao matemático hindu Brahmagupta.O seu conhecimento é de extrema importância para resolu-ção de problemas olímpicos.

5. (Treinamento Olímpico Brasileiro) Seja AD a bissetriz do ângulo A do triângulo ABC. Sejam M e N os pontossobre as semi-retas AB e AC respectivamente e P o ponto de interseção de AD com MN. Prove que:

Se ∠ MDA = ∠ ABC e ∠ NDA = ∠ BCA, então AD— 3 = AB— ⋅ AC— ⋅ AP—

➲ atividade de casa1. (Olimpíada Americana) ABC é um triângulo eqüilátero

inscrito em uma circunferência como mostra-se na fi-gura ao lado. Um ponto P é escolhido sobre o arco BCe os segmentos AP, BP e CP são traçados com PB— = 5e PC— = 20. Se AP intercepta BC no ponto D, qual é ocomprimento do segmento AD?

2. a) (Olimpíada Russa) Diagonais AC e BD do quadrilátero ABCD convexo encontram-se num ponto O. Prove que:Se AB— = OD—, AD— = CO— e ∠ BAC = ∠ BDA, então ABCD é um trapézio.

b) (Olimpíada do Reino Unido) Seja ABCD um retângulo, P ponto médio de AB, Q um ponto sobre a reta PD talque CQ é perpendicular a PD. Prove que o triângulo BQC é isósceles.

c) (OBM/2000) O trapézio ABCD tem bases AB e CD. O lado DA mede x e o lado BC mede 2x. A soma dosângulos DÂB e ABC é 120º. Determine o ângulo DÂB.

OP AD= ⋅12

.

CDAD

.

xCB

A

D

E(figura 1)

A

B

CM(figura 2)

T

A

D

E

B CF

TE perpendicular a ADF ponto médio de BC

O

A

B

CP

Page 33: obmnivel3b

SISTEMA ANGLO DE ENSINO 33 ANGLO VESTIBULARES

3. a) (Treinamento Olímpico Colombiano) Sejam S1 e S2 duas circunferências que se intersectam em dois pontos

distintos P e Q. Sejam l1 e l2 duas retas paralelas, tais que:

i. l1 passa por P e intersecta S1 em um ponto A1 distinto de P e a S2 em A2 distinto de P.

ii. l2 passa por Q e intersecta S2 em um ponto B1 distinto de Q e a S2 em B2 distinto de Q.

Demonstrar que os triângulos A1 QA2 e B1 P B2 tem o mesmo perímetro.

b) (Olimpiada de Leningrado) Sejam K e L respectivamente pontos sobre os lados AB e CD de um trapézio (combases AD e BC). Prove que: se ∠ BAL = ∠ CDK, então ∠ BLA = ∠ CKD

4. a) (OBM) Uma reta que passa pelos pontos médios de dois lados opostos de um quadrilátero convexo forma ân-gulos iguais com ambas as diagonais. Mostre que as duas diagonais têm o mesmo comprimento

b) (OBM) Em um quadrilátero convexo ABCD, os lados opostos AD e BC são congruentes e os pontos médios dasdiagonais AC e BD são distintos. Prove que a reta determinada pelos pontos médios das diagonais formaângulos iguais com AD e BC.

5. (Olimpíada Americana) ABCD é um quadrilátero cíclico. Prolongam-se os lados AB e DC até se encontrarem numponto E, e os lados AD e BD até se encontrarem em um ponto F. Mostre que as bissetrizes dos ângulos E e F sãoperpendiculares.

6. (Olimpíada do Reino Unido) Seja ABCD um quadrilátero cíclico com ∠ A = 90º. Considere ainda que AB e BC sãoos menores lados do quadrilátero. Sejam P e Q pontos em AD e CD tais que AP

—= AB

—e BC

—= CQ

—. Prove que o

ponto médio de PQ pertence a mediatriz de AC.

Page 34: obmnivel3b

SISTEMA ANGLO DE ENSINO 34 ANGLO VESTIBULARES

AULAS 1 e 2Resoluções das Atividades de Aula1. Do vértice A do quadrado podemos escrever:

∠ DAE + ∠ EAF + ∠ FAB = 90°Desde que ∠ EAF = 45°, seque-se que:

∠ DAE + ∠ FAB = 45°Prolongando-se o segmento CD (de C para D) conforme figura e

marcando sobre este um ponto G tal que GD—

= FB— —

DG = BF—

,obtém-se os triângulos ADG e ABF são congruentes (LAL).

Desta congruência, resulta AG—

= AF—

e ∠ GAD = ∠ FAB.Além disso, podemos também escrever: ∠ GAE = ∠ GAD + ∠ DAE = ∠ FAB + ∠ DAE = 45° = ∠ EAF

que implica juntamente com AG—

= AF—

e o lado AE (comum) que os triângulos GAE e EAF são também congruentes. En-

tão como conseqüência, GE—

= EF—

.

O perímetro do triângulo CEF é dado pela soma: EC—

+ CF—

+ EF—

Por outro lado, EC—

+ CF—

+ EF—

= EF—

+ EC—

+ CF—

= GE—

+ EC—

+ CF—

= (GD—

+ DE—

) + EC—

+ CF—

= (FB—

+ DE—

) + EC—

+ CF—

= DE—

+ EC—

+ (CF—

+ FB—

)

= (DE—

+ EC—

) + (CF—

+ FB—

)

= DC—

+ CB—

= 2Resposta: O perímetro do triângulo CEF é 2.

2. a) Usando a propriedade da desigualdade triangular nos triângulos ABD, BCD, ABC e CDA

obtém-se respectivamente as desigualdades:

Da adição membro a membro obtém-se AB—

+ BC—

+ CD—

+ DA—

� AC—

+ BD—

b) Usando as propriedades: • Oposto ao maior lado de um triângulo está o ângulo de maior medida (vice-versa).• Ângulos opostos pelo vértice são congruentes.Decorre do enunciado a seqüência de desigualdades entre as medidas de ângulos internos dos triângulos: BAC,CDE, EFG, GHI e IKA respectivamente:

∠ BAC � ∠ BCA ( = ∠ DCE)∠ DCE � ∠ DEC ( = ∠ FEG)∠ FEG � ∠ FGE ( = ∠ HGI)∠ HGI � ∠ HIG ( = ∠ KIA)∠ KIA � ∠ KAI ( = ∠ BAC)

Desta Seqüência, obtém-se ∠ BAC � ∠ BAC, o que é um absurdo.Portanto, nestas condições, é impossível construir a estrela.

+

+

+

+

+

AB AD BD

BC CD BD

AB BC AC

AD CD AC

ResoluçõesNÍVEL 3

A B

C

F

EDG

45°

Page 35: obmnivel3b

SISTEMA ANGLO DE ENSINO 35 ANGLO VESTIBULARES

3. a) No octógono regular: AE, BF, CG e HD são diâmetros de comprimento igual a 1.Assim, os triângulos: PAE, PBF, PCG e PHD estão inscritos em uma semicircunferência, conseqüentemente são re-tângulos em P respectivamente. Nestas condições, decorre do teorema de Pitágoras:

Adicionando-se membro a membro, resulta:

PA— 2 + PB

— 2 + PC— 2 + PD

— 2 + PE— 2 + PF

— 2 + PG— 2 + PH

— 2 = 4

b) Usando raciocínio análogo ao que foi feito no item (a) encontramos:

4. Seja M o pé da altura relativa ao lado AB.Seja N o pé da altura relativa ao lado AC.Desde que o triângulo ABC é acutângulo estespés pertencem aos lados do triângulo e os pontosP e Q pertencem aos semicírculos de diâmetrosAB e AC respectivamente.Conseqüentemente, os triângulos APB e AQC sãoretângulos em P e Q respectivamente. (figura).

Por outro lado,

Das relações métricas do triângulo retângulo, temos: [1]

Os triângulos retângulos ∆ ANB e ∆ AMC com um ângulo  (∠ A � 90°), em comum, são semelhantes

pelo caso (∼ AA), daí: , ou AN—

⋅ AC—

= AM—

⋅ —AB [2]

Portanto, de [1] e [2], conclui-se que AP—

= AQ—

, o que finaliza a demonstração.

5. Usando o teorema dos senos:

a = 2R senA, b = 2R senB e c = 2R senC

R é o raio da circunferência circunscrita ao triângulo ABC.

Utilizando o teorema dos cossenos, obtemos:

Simplificando, obtém-se

O que finalizando a demonstração.

sen2A = sen2B + sen2C – 2 ⋅ senB ⋅ senC ⋅ cosAsen2B = sen2C + sen2A – 2 ⋅ senC ⋅ senA ⋅ cosBsen2C = sen2A + sen2B – 2 ⋅ senA ⋅ senB ⋅ cosC

4R2 ⋅ sen2A = 4R2 ⋅ sen2B + 4R2 ⋅ sen2C – 2 ⋅ 4R2 ⋅ senB ⋅ senC ⋅ cosA4R2 ⋅ sen2B = 4R2 ⋅ sen2C + 4R2 ⋅ sen2A – 2 ⋅ 4R2 ⋅ senC ⋅ senA ⋅ cosB4R2 ⋅ sen2C = 4R2 ⋅ sen2A + 4R2 ⋅ sen2B – 2 ⋅ 4R2 ⋅ senA ⋅ senB ⋅ cosC

AN

AM

AB

AC=

AP AM AB

AQ AN AC

2

2

= ⋅

= ⋅

PAn

Sn

ii

n 2

1 2 2=∑ = ∴ = .

+

+ =

+ =

+ =

+ =

PA PE

PB PF

PC PG

PH PD

2 2

2 2

2 2

2 2

1

1

1

1

A

BC

D

E

FG

HP

A

P

M

B C

Q

N

H

A

B

C

a

c

b

R

Page 36: obmnivel3b

SISTEMA ANGLO DE ENSINO 36 ANGLO VESTIBULARES

6. a) B, P e C são colineares. Decorre então ∠ BPA + ∠ APC = 180°.Daí, cos ∠ APB + cos ∠ APC = 0, ou seja cos ∠ APB = –cos ∠ APCPor outro lado, usando este resultado e o teorema dos cossenos nos triângulos APB e APC:

Multiplicando a primeira relação por n e a segunda por m, em seguida adicionando-as membro a membro,encontramos:

nc2 + mb2 = (n + m)p2 + nm(m + n)

Como n + m = a, segue-se do resultado acima a igualdade: a ⋅ (p2 + m ⋅ n) = b2 ⋅ m + c2 ⋅ nFinalizando a demonstração do Teorema de Stewart

b) (resolução pelo teorema dos senos) Do triângulo ABC: ∠ ACB + ∠ CBA + ∠ BAC = 180° ∴ ∠ ACB + 40° + 40° = 180°

∴ ∠ ACB = 100°. Daí, ∠ CPB = ∠ ACB – ∠ PCA = 100° – 20° = 80°Assim, fazendo ∠ PBA = α temos ∠ PBC = 40° – α.Aplicando o teorema dos senos no triângulo:

∆APC:

∆APB:

∆CPB:

Daqui para frente, com raciocínio análogo ao que foi feito pelo item (a), chegaremos em

Donde obtém-se Por inspeção, constatamos que α = 20°. Esta solução é única; pois a função f dada pela lei

f(x) = e definida para qualquer a e 0 � x � é injetora (*).

Então, ∠ PBC = 40° – α = 20° Portanto, do triângulo CPB, temos que:

∠ BPC = 180° – ∠ BCP – ∠ PBC = 100° – 20° = 80°Nota (*): Justificativa

(∀α )(∀β ): f(α) = f(β) ⇔

⇔ sena ⋅ cotgα – cosa = sena ⋅ cotgβ – cosa⇔ cotgα = ⋅ cotgβ⇔ α = β

Resoluções da Tarefa de Casa1. a) Seja ∠ EDF = ∠ FDC = α.

Seja F’ um ponto sobre o prolongamento do lado BA (no sentido de B para A). Assim, decorre do enunciado que: • ∠ F’ED = ∠ EDF + ∠ FDC = 2 ⋅ α (alternos e interno) [1]

• Os triângulos retângulos FCD e F’AD são congruentes, pois

CD—

= DA—

= l e FC—

= F’A—

= y. Daí, ∠ F’DA = ∠ FDC = α [2]

sena sen asen

sena sen asen

⋅ ⋅ = ⋅ ⋅cos – cos cos – cosα αα

β ββ

sen asen

sen asen

( – ) ( )αα

ββ

= −

π2

sen a xsenx( )−

sensen

sensen

( – )40 2020

° = °°

αα

PBsen

PCsen80 40°

=°( – )α

PAsen

PBsenα

=°30

PCsen

PAsen10 20°

c2 = p2 + m2 + 2mpcos ∠ APCb2 = p2 + n2 – 2npcos ∠ APC

10°

30°

20° 80°

40° – α

αA B

C

P

A B

CD

E

F

y

yF’ x

α αα

Page 37: obmnivel3b

SISTEMA ANGLO DE ENSINO 37 ANGLO VESTIBULARES

Por outro lado, de [1] e [2] podemos escrever:∠ EF´D = 90° – ∠ F’DA = 90° – α. como conseqüência,∠ EDF´ = 180° – (∠ F’ED + ∠ EF’D) = 180° – (2 ⋅ α + 90° – α) = 90° – α.

Nestas condições, ∠ EF’D = ∠ EDF’= 90° – α, o que implica, dizer que o triângulo F’ED é isósceles de base F’D.

Portanto, F’E—

= x + y = DE—

= a

b) Sejam u,v e w medidas em graus.Sejam A, B, C, D, E, F e I pontos como indicados na fi-gura ao lado. Nestas condições, decorre dos quadriláte-ros inscritíveis AEID, EBFI e FCDI (cíclicos):

∠ DIE = 180° – u; ∠ EIF = 180° – v e ∠ FID = 180° – w Daí: 180° – u + 180° – v + 180° – w = 360°Portanto, u + v + w = 180°

2. a) Sejam E e F pontos sobre os segmentos DB e AD respectivamente tais que: DF—

= DE—

= EB—

= 1Desta construção e dos dados do enunciado, temos sucessivamente:

O triângulo FDC é isósceles de base FC. Daí, ∠ DFC = ∠ DCF = ⋅ ∠ ADB = 30º. [1]

O triângulo FDE é eqüilátero. Daí, ∠ ADB = ∠ FDE = ∠ DFE = ∠ FED = 60º. [2]

O triângulo FDB é isósceles de base BF. Daí, ∠ FBE = ∠ BFE= [3]

como mostra na figura abaixo:

Por outro lado,

Desde que ∠ ACD = 45º, tem-se de [1]:

∠ FCD = 30ºº e ∠ FCA = 15°º

Como conseqüência deste resultado, [2] e [3]:∠ BFD = ∠ BFE + ∠ DFE = 30° + 60° = 90°.∆AFC é isósceles onde ∠ FAC = ∠ FCA = 15º e CF = AF.

∆BFC é isósceles onde ∠ FBC= ∠ FCB = 30º e CF—

= BF—

.

Nestas condições, o triângulo AFB é isósceles e

∠ FAB = ∠ FBA =

Portanto, ∠ ABC = ∠ FAB + ∠ FBC = 45ºº + 30º = 75º

1802

45° − ∠ = °BFD

B

A C15°

45°

45°30°

120°60°

30°

60°30°

60°

30°15°

1

1

1

D

E

F

1

1802

180 1202

30° − ∠ = ° = °BEF –

12

v

u

w

E

A

D

CF

B

I

Page 38: obmnivel3b

SISTEMA ANGLO DE ENSINO 38 ANGLO VESTIBULARES

b) Do enunciado, tem-se no triângulo ABC:

 + B + C = 180° ∴ 20° + 120° + C = 180° ∴ C = 40° [1]

Por outro lado, seja M simétrico de B em relação a reta suporte de AQ.Como ∠ QAB = ∠ QAC = 10°, então M pertence ao segmento AC. Marcando este ponto sobre AC e ligando-o a Q e a B por segmentos de reta obtém-se a figura:Como conseqüência desta simetria, resulta:

AB—

= AM—

, QB—

= QM—

[2]e ∠ QMA = ∠ QBA = 20° [3]

De [2], segue-se que os triângulos:BAM e BQM são isósceles. [4]

Desde que ∠ A = 10° + 10° = 20° e∠ ABC = ∠ ABQ + ∠ QBC = 20° + 100° = 120°,

conclui-se de [3]:

∠ ABM = ∠ AMB = ⋅ (180° – 20°) = 80°.

como conseqüência ∠ MBC = ∠ ABC – ∠ ABM = 120° – 80° = 40° [5]

Assim, de [4], ∠ QBM = ∠ QMB = 80° – 20° = 60°. Logo, o triângulo BQM é eqüilátero.

O que implica QB—

= QM—

= MB—

[6]

Portanto, de [1], [5] e [6], conclui-se que QM—

= MB—

= MC—

. Nestas condições, o triângulo QMC é isósceles e∠ QMA = 20° (medida do ângulo externo relativa ao vértice M do triângulo QMC),

conseqüentemente ∠ ACQ =

segunda resolução (teorema dos Senos)Do triângulo ABC: ∠ ACB + ∠ CBA + ∠ BAC = 180° ∴ ∠ ACB + 120° + 20° = 180° ∴ ∠ ACB = 40°.Assim, fazendo ∠ ACQ = x temos ∠ QCB = 40° – x.Aplicando o teorema dos senos no triângulo:

Multiplicando membro estas relações e em seguida simplificando, encontramos:

Lembrando que sen100° = sen80° = cos10°; sen20° = 2 ⋅ sen10° ⋅ cos10° e sen30° =

têm-se: ∴ x = 10° (*)

Por inspeção, constatamos que x = 10°. Esta solução é única; pois a função f dada pela lei

f(x) = e definida para qualquer a e 0 � x � é injetora.

Justificativa:

(∀α )(∀β) : f(α) = f(β) ⇔

⇔ sena ⋅ cotgα – cosa = sena ⋅ cotgβ – cos a⇔ cotgα = cotgβ⇔ α = β

sena sen asen

sena sen asen

⋅ ⋅−= ⋅ − ⋅cos cos cos cosα α

αβ β

β

sensen

sen asen

a( ) ( )−= −α

αβ

β

π2

sen a xsenx( )−

sen xsenx sen

sensen

sen xsenx

sensen

( – ) ( )40 12 10

3010

40 3010

° =⋅ °

= °°

° − = °°

12

110 100

120 10 40sen senx sen sen sen sen x° °

=° ⋅ ° °⋅ ⋅ ⋅ ( – )

∆ ∆ ∆AQB

QBsen

QAsen

AQCQA

senxQC

senCQB

QCsen

QBsen x

: ; : ; :( )10 20 10 100 40°

=° °

=° −

∠ = ° = °QMA2

202

10

12

10°10°

20°60°

60°20° 40°

Q

B

A MC

Page 39: obmnivel3b

SISTEMA ANGLO DE ENSINO 39 ANGLO VESTIBULARES

c) Seja X um ponto sobre o lado BC tal que PB—

= BX—

.Daí, segue-se que o triângulo BPX é isósceles. Assim, como conseqüência desta construção e do enun-ciado, obtém-se a figura auxiliar ao lado, onde

∠ CPX = 180° – (60° + 80°) = 40°daí ∆PXC isósceles com XC = XP.

Nestas condições, a prova de que: BC—

= AP—

+ PB—

é

decorrente apenas da prova de AP—

= XP—

.Por outro lado, do teorema dos senos, resulta dos triângulos:

∆ABP: e ∆CXP:

Donde conclui-se que , isto é, AP—

= XP—

Portanto, este último resultado permite afirmarmos que: BC—

= AP—

+ PB—

, finalizando a demonstração.

3. a) Seja N ponto médio de AM, isto é, NA—

= NM—

Desde que ∠ SAB = 45°, o triângulo retângulo ASM é isósceles e∠ SMA = 45°. Nestas condições, o segmento SN é medianacomo também altura, relativa a hipotenusa AM, isto é, SN éperpendicular a AM. Daí,

SN—

= AN—

= NM—

= ⋅ AB—

[1]

Os triângulos CAB e SNB são semelhantes (SN // CA) [2]

Portanto, de [1] e [2] resulta:

b) Desde que AC’ = AB’ = 1, têm-se que o triângulo AB´C´ é isósceles eretângulo em A. Ainda como conseqüência,

∠ AB’C’ = 45° e B’C’ = [1]

Sejam C” e B” as projeções ortogonais de C´ e B´ respectivamente so-bre a hipotenusa BC. Desde que CC´ e BB´ são bissetrizes têm-se que

C’C’’

= C’A

1 e B’B”

= B’A

= 1.Daí, a reta B´C´ é paralela a reta CB.Então ∠ ACB = ∠ ABC = 45° [2]

De [1] e [2]; B”C’’

= B’C’

= e BC’’

= B’’C

= 1

Portanto,

c) Desde que ABC eqüilátero e PA // BS, então

∠ PAQ = ∠ ABC = 60° [1]e ∆PAR ~ ∆SCR [2]

Como, PQ—

= QR—

= RS—

, resulta

De [1]: PAR é um triângulo isósceles com PA—

= AR—

, [3](pois, AQ é simultaneamente mediana e bissetriz, relativaao lado PR.)

De [2]: , daí [4]

Portanto, de [3] e [4], obtém-se SC—

= 1.

AR RC AR AC e SC

PA= ⋅ ∴ = = =213

12

PA

SC

AR

RC

PR

SR

RSSR

= = = ⋅ =22

BC BC C B B C= + + = + + = +” ” ” ” 1 2 1 2 2

2

2

SN

CA

NB

AB

AB NB

AB

AB

ABAB= = =∴ ∴

14

3

34 3

14

AP

BP

XP

BP=

XP

BP

sensen

= °°

2080

AP

BP

sensen

sensen

= °°

= °°

20100

2080

20°20°

100°

A

60°

80° 40°

40°80°B Cx

P

45°45°

MNA

C

3

S

B

45° 45°

C’

C’

B’’

B’A

B

C

1

1

1

1

1

1

60°60°

60°

R

SCB

AP

Q

Page 40: obmnivel3b

SISTEMA ANGLO DE ENSINO 40 ANGLO VESTIBULARES

4. Desde que ABCD é um quadrado, tem-se.

AB—

= BC—

= CD—

= DA—

= 2 ⋅ MA—

[1]Seja α = ∠ ABM. (α em graus)Seja F a intersecção da reta DP com o lado AB.Desde que DC // AB, têm-se que os triângulos EPF e CPD são semelhantes.

Então: [2]

Por outro lado,

mas de [1], têm-se tgα = . Assim, = [3]

Portanto, de [1], [2] e [3], encontramos

O que equivale dizer que a reta DP divide o segmento EB em dois segmentos de mesma medida.

Resolução OficialProlongue BM até encontrar o prolongamento do lado CD no pontoN. Claramente, ∆AMB ≡ ∆DMN, donde segue que AB = DN. Por-tanto, D é o ponto médio de CN. O resultado segue observando queos triângulos CPN e EPB são semelhantes, e como PD é medianado triângulo CPN, conclui-se que o prolongamento de DP encontraEB em seu ponto médio.

5. a) Usando a propriedade da desigualdade triangular nos triângulos ABD, BCD, ABC e CDA obtém-se respec-tivamente as desigualdades:

Daí, AB—

+ BC—

+ CD—

+ DA—

� AC—

+ BD—

.

Logo: [1]

Por outro lado, usando a propriedade da desigualdade triangular nos triângulos ABO, BOC, COD e DOA obtém-serespectivamente as desigualdades:

Adicionando membro a membro, obtém-se AB—

+ BC—

+ CD—

+ DA—

� AO—

+ OC—

+ BO—

+ OD—

= 2 ⋅ (AC—

+ BD—

)

Daí, podemos escrever � 2 [2]

Portanto, de [1] e [2], encontramos: 1 � � 2AB BC CD DA

AC BC

+ + ++

AB BC CD DA

AC BC

+ + ++

+

+

+

+

+

AO BO AB

BO OC BC

OC OD CD

OD AO DA

1�AB BC CD DA

AC BC

+ + ++

+

+

+

+

+

AB AD BD

BC CD BD

AB BC AC

AD CD AC

EF

CD

EB

BCEB EB=

⋅=⋅ ⋅

22

EP

PC

EB

BCtg

EBBC

= =⋅⋅

α2

MA

AB

MA

MA=

⋅=

2

12

EP

PC

EBPCBP

EB PB sen

BC PB sen

EB

BCtg= =

⋅ ⋅ ⋅

⋅ ⋅ ⋅ ° −= ⋅( )

( )( )

12

12

90

αα

α

EF

CD

EP

PC=

D

A B

C

M

E F

P

A B

D CN

MP

E

A

B

C

DO

Page 41: obmnivel3b

SISTEMA ANGLO DE ENSINO 41 ANGLO VESTIBULARES

b) Sejam a, b, c ,d comprimentos dos lados.Sejam p e q comprimentos das diagonais.Seja α a medida do ângulo formado pelas diagonais e O a intersecçãodestas.Desde que o quadrilátero é convexo. Traçando paralelas por cada vérticea diagonal que não passa por ele, construímos um paralelogramo de área2 ( dobro da do quadrilátero), lados de comprimentos p e q, e com α sen-do a medida de um dos ângulos internos.Nestas condições, da área do paralelogramo, podemos escrever:

2 = p ⋅ q senα � p ⋅ q ∴ p ⋅ q � 2Como conseqüência, (p + q)2 = (p – q)2 + 4pq � 4pq � 8.

donde [1]

Por outro lado, para os lados do quadrilátero podemos escrever:

• 1 � , decorrente da soma das áreas dos dois triângulos em que o quadrilátero fica decomposto

através da diagonal de comprimento q.

• 1 � , decorrente da soma das áreas dos dois triângulos em que o quadrilátero fica decomposto

através da diagonal de comprimento p.

destas desigualdades obtém-se: ab + bc + cd + da � 4, ou ainda (a + c) (b + d) � 4.

Daí, (a + b + c + d)2 = ((a + c) – (b + d))2 + 4(a + c) (b + d) � 4 (a + c) (b + d) � 16, donde conclui-se que: a + b + c + d � 4 [2]

Portanto, de [1] e [2], resulta: a + b + c + d + p + q � 4 + 2

6. Suponhamos que BC—

� AB—

(veja figura) o que não perde em generalidade,

Tomemos sobre os lados BC e BG pontos K e L respectivamente tais que BK—

= BL—

= AB—

.

Em seguida tracemos as bissetrizes BM e BN dos ângulos, ABC e D BG, respectivamente.Desde que ∠ ABC + ∠ DBG = 360° – ∠ ABD – ∠ CBG = 360° – 90° – 90°, temos que ∠ ABC + ∠ DBG = 180°,∠ MBK + ∠ NBL = 180° e ∠ MBN = ∠ MBK + ∠ NBL + ∠ KBL = 180°, donde conclui-se que M, B e N são pontoscolineares.Por outro lado, desde que BM ⊥ DL (decorrente da construção acima), então AK é paralelo a DL.Assim, da condição AC // DG, obtemos que: ∠ BKA = ∠ BCA e ∠ BLD = ∠ BGD, o que implica dizer que o triân-gulo ABC é isósceles; finalizando a demonstração pedida.

7. Desde que AP é mediana, têm-se m = n = . Então do teorema de Stewart:

c2 ⋅ + b2 ⋅ = a ⋅ donde resulta b2 + c2 = 2 ⋅

pa2

2

2+

pa a2

2 2+ ⋅

a2

a2

a2

A

B

C

D

E

G

F

N

MK

L

2 2 2 2= +( )

bc da2 2

+

ab cd2 2

+

p q+ =� 8 2 2

a

bc

d

q

p

Page 42: obmnivel3b

SISTEMA ANGLO DE ENSINO 42 ANGLO VESTIBULARES

AULAS 3 e 4Resoluções das Atividades de Aula

1. a) Aplicando o teorema de Ceva no triângulo ABC, temos:

Por outro lado, os triângulos ABD e ABC tem a mesma altura em relação ao lado BC, então:

b) Aplicando o teorema de Menelaus no triângulo ABC, em relação a secante DFE, temos:

Por outro lado, os triângulos DFB e DFC tem a mesma altura em relação a reta BC, então:

2. a) Da figura acima e o recíproco de CEVA, temos:

⇒ terna: (AD, BE, CF) de cevianas não concorrentes.

⇒ terna: (AD’, BE’, CF’) de cevianas concorrentes.

b) Da figura dada e o recíproco de Menelaus, temos:

⇒ terna: (D, E, F) de pontos não colineares.

⇒ terna: (D’, E’, F’) de pontos colineares.

3. Sejam AL, BM e CN bissetrizes internas relativas aos lados BC—–

= a, CA—–

= b e AB—–

= c respectivamente de um triân-gulo ABC. Decorre do teorema da bissetriz interna:

e . Daí, . Portanto, como conseqüência do recíproco do teorema de

CEVA (T2), conclui-se que as cevianas AL, BM e CN (bissetrizes) são concorrentes.

4. Desde que MQ e MP são bissetrizes internas dos triângulos AMCe BMC respectivamente, resulta do teorema da bissetriz interna:

Multiplicando membro a membro estas igualdades, obtém-se:

Daí, . Portanto, pelo recíproco do teorema de CEVA, conclui-se que as cevianas AP, BQ e CM do

triângulo ABC são concorrentes.

AM

MB

BP

PC

CQ

QA⋅ ⋅ = 1

CQ

QA

BP

PC

CM

AM

MB

CM⋅ ⋅=

CQ

QA

CM

AMe

BP

PC

MB

CM= =

BL

LC

CM

MA

AN

NB⋅ ⋅ = 1

AN

NB

ba

=BL

LC

cb

CM

MA

ac

= =,

AF

F B

BD

D C

CE

E A

’⋅ ⋅ ⋅ ⋅= =1

361

12

1

AF

FB

BD

DC

CE

EA⋅ ⋅ ⋅ ⋅= = ≠1

523

21

415

1

AF

F B

BD

D C

CE

E A

’⋅ ⋅ ⋅ ⋅= =1

223

31

1

AF

FB

BD

DC

CE

EA⋅ ⋅ ⋅ ⋅= = ≠2

132

22

3 1

( )( )DFBDFC

BD

DC= = 1

3

BD

DC

CE

EA

AF

FB

BD

DC

BD

DC⋅ ⋅ ∴ ⋅ ⋅ ∴= = =1

42

32

113

( )( )ABDABC

BD

BC

BC

BD DC= =

+= 4

7

BD

DC

CE

EA

AF

FB

BD

DC

BD

DC⋅ ⋅ ∴ ⋅ ⋅ ∴= = =1

24

32

143

A

C

BM

QP

Page 43: obmnivel3b

SISTEMA ANGLO DE ENSINO 43 ANGLO VESTIBULARES

5. Desde que BM é mediana e (ABC) = 100:

Desde que AB—–

= 10, AC—–

= 30 e AP é bissetriz do ânguloBÂM podemos escrever

6. Desde que os triângulos BKC e CHB são obviamente congruentes, temos CK—–

= HB—–

e BK perpendicular a KC. LogoBK é uma altura, e ABC é isósceles, com AB = BC. Além disso, ∠ HBC = ∠ KCB; portanto AB

—–= AC

—–, o que finaliza a

demonstração.

Resoluções da Tarefa de Casa

1. a) Desde que BC—–

= 8, têm-se DC—–

= 8 – BD—–

= 8 – x. Por outro lado, CF é mediana. Então ; BE é bissetriz.

Então . Nestas condições, aplicando teorema de CEVA no triângulo ABC, resulta:

. Daí obtém-se .

b) Desde que BC—–

= 8, DC—–

= 2, CA—–

= 7 e EA—

= y, têm-se BD—–

= 10 e CE—–

= 7 – y.

Por outro lado, F é ponto médio de AB. Então . Nestas condições, aplicando teorema de MENELAUS no

triângulo ABC, secante DEF, resulta: Daí, .

2. a) Inicialmente, observe que:

∴ BD—–

= 1 ⋅ k e DC—–

= 2 ⋅ k (k � 0)

∴ CE—–

= 1 ⋅ n e EA—

= 2 ⋅ n (n � 0)

Aplicando o teorema de Menelaus no triângulo ADC, segundo a secante BZE, temos:

Aplicando o teorema de Menelaus no triângulo BEC, segundo a secante AZD, temos:

b) Aplicando o teorema de Pitágoras no ∆ABC:

BC—–2 = AB

—–2 + AC—–2 ∴ BC

—–2 = 9 + 16 = 25 ∴ BC—–

= 5

Desde que BC—–

= 5 e BL—

= x, têm-se LC—–

= 5 – xPortanto, aplicando o teorema de Ceva, obtém-se:

BL

LC

CM

MA

AN

NB

xx

x x x⋅ ⋅ ∴ ⋅ ⋅ ∴ ∴= = = =15

22

21

1 2 553–

EZ

ZB

BD

DC

CA

AE

EZ

ZB

kk

n nn

EZ

ZB

EZ

ZB⋅ ⋅ ∴ ⋅ ⋅ ⋅ ⋅

⋅∴ ⋅ ⋅ ∴= + = = =1

12

1 22

112

32

143

AZ

ZD

DB

BC

CE

EA

AZ

ZD

kk k

AZ

ZD

AZ

ZD⋅ ⋅ ∴ ⋅ ⋅ ∴ ⋅ ⋅ ∴=

+= = =1

11 2

12

113

12

1 6

CE

EA= 1

2

BD

DC= 1

2

y = 356

AF

FB

BD

DC

CE

EA

yy

⋅ ⋅ ⋅ ⋅= =33

102

71

–.

AF

FB= =3

31

x = 247

AF

FB

BD

DC

CE

EA

xx

⋅ ⋅ ⋅ ⋅= =33 8

43

1–

CE

EA= =8

643

AF

FB= =3

31

∴ ⋅= =( ) ( )ABP ABM1025

20

( )( )ABPABM

BP

BM

AB

AB AC= =

+=

12

1025

( ) ( )ABM ABC= =⋅12

50

C

A

BD

P

M

A

B C

N

2

1O

M

Lx

Page 44: obmnivel3b

SISTEMA ANGLO DE ENSINO 44 ANGLO VESTIBULARES

3. a) Sejam L, M e N pontos médios dos lados BC, CA e AB respectivamente. Então, . Daí,

. Portanto, como conseqüência do recíproco do teorema de CEVA (T2), concluímos que as cevia-

nas AL , BM e CN (medianas) são concorrentes,

b) É fácil verificar que o ponto médio inverte a razão correspondente, isto é, .

Daí, podemos escrever:

Por outro lado, do teorema de CEVA e da concorrência das cevianas AD, BE e CF têm-se .

Portanto, da igualdade acima, resulta , o que implica pelo teorema recíproco de CEVA (T2) que

as cevianas AD’, BE’ e CF’ são também concorrentes.

4. Seja L o lado do hexágono regular.

Seja AC = CE = l (comprimento da diagonal AC).Seja G a interseção das retas EC e AB.

Nestas condições,

• (comprimento do lado do hexágono)

• AM—–

= CN—–

= λ ⋅ l, e como conseqüência MC—–

= NE—–

= l(1 – λ)• No triângulo ACG temos ∠ A = 30° e ∠ C = 120°.

Daí, ∠ G = 30°. Conseqüentemente ACG é isósceles, com

.

Por outro lado,

AG—–

= AB—–

+ BG—–

= L + BG—–

= 3 ⋅ L ∴ BG—–

= 2 ⋅ L e NG—–

= NC—–

+ CG—–

= λ ⋅ l + l = l ⋅ (1 + λ)Então, aplicando o teorema de Menelaus no triângulo ACG, tendo a reta BMN como transversal aos seus lados,encontramos:

5. Desde que e , da identidade:

, obtém-se:

[1]

Das igualdades : e têm-se

[2]DB

AB

EC

BCe

AF

CA= = =1 1 1– , – –α β γ

CF

CA= γ

AD

AB

BE

BC= =α β,

αβ αγ βγ+ + = 145

( ) ( )α β γ α β γ αβ αγ βγ+ + = + + + ⋅ + +2 2 2 2 2

α β γ2 2 2 25

+ + =α β γ+ + = 23

CN

NG

GB

BA

AM

MC

LL

⋅ ⋅ ∴ ⋅⋅

⋅ ⋅ ⋅ ⋅⋅

∴ ∴=+

= = =11

21

12

11

33

2

2

λλ

λλ

λλ

λl

l

l

l( ) ( – ) –

AC CG e AG L L= = = = = =⋅ ⋅l l l2 30 3 3 32cos º ( )

AB BC L= = =l

3

AF

F B

BD

D C

CE

E A

’⋅ ⋅ = 1

AF

FB

BD

DC

CE

EA⋅ ⋅ = 1

AF

F B

BD

D C

CE

E A

AF

FB

BD

DC

CE

EA

’⋅ ⋅ ⋅ ⋅ ⋅

= 1

BD

D C BD

DC

CE

E A CE

EA

eAF

F B AF

FB

’,

’=

=

=

1 1 1

BL

LC

CM

MA

AN

NB⋅ ⋅ = 1

BL

LC

CM

MA

AN

NB= = =1

30°

30°

2 ⋅ L

G

λ1

CLB

L

A

L

F EL

L

D

1 (1 – λ)M

λ1

λ1

N

A

F

CEB

D

Page 45: obmnivel3b

SISTEMA ANGLO DE ENSINO 45 ANGLO VESTIBULARES

Usando a propriedade: “áreas de triângulos de alturas de comprimentos iguais são proporcionais aos comprimentosdas correspondentes bases destes triângulos”, [1] e [2], podemos escrever sucessivamente:

[3]

[4]

[5]

Por outro lado, (DEF) + (ADF) + (BED) + (CFE) = (ABC) (ver figura)

Ou ainda

Portanto, da igualdade acima e as razões obtidas em [3], [4] e [5] encontramos:

6. a) (resolução Oficial) Seja M o ponto médio de BC e H o pé da altura relativa a A. Temos que AH é comum aos tri-ângulos AHM e AHB, AHB ≅ AHM (retos) e HÂM ≅ HÂB, logo, pelo caso ALA, os triângulos AHM e AHB sãocongruentes.

Assim, , pois MC—–

= MB—–

.

Desde que AM é bissetriz de HÂC, pelo teorema das bissetrizes

Como 0 � 2α � 180° e 2α = 60° então α = 30°.

Portanto, os ângulos do triângulo ABC, são ∠ BAC = 3α = 90°, ∠ ABC = 90° – α = 60° e ∠ ACB = 90° – 2α = 30°

b) Sejam BC—–

= a, AC—–

= b e AB—–

= c

Utilizaremos a figura ao lado como referência.

Dividiremos a nossa resolução em três casos:

Primeira Caso: P é um ponto do lado AC.Neste caso, AP + CP = b. (constante) enquanto BPatingi o seu mínimo quando P = A, caso contrário,BP será a hipotenusa do triângulo retângulo ABP econseqüentemente, BP � BA. Portanto, neste caso, o mínimo de AP + BP + CP éb + c e ocorrerá quando P = A.

Segundo Caso: P é um ponto do lado ABRaciocinando de modo similar ao que fizemos no primeiro caso, concluiremos que o valor mínimo de AP + BP + CPé b + c e também ocorrerá quando P = A.

Terceiro Caso: P é um ponto do lado BCNeste caso, BP + CP = a (constante) enquanto AP atingi o seu mínimo quando P coincide com a projeção ortogonalde A sobre o lado BC. Isto é, quando AP é perpendicular a BC. Neste caso, fazendo AP = h ( altura do triânguloABC), o valor mínimo de AP + BP + CP é a + h.Por outro lado, a + h � b + c, pois

(a + h)2 = a2 + 2ah + h2 = (b2 + c2) + 2bc + h2 = (b + c)2 + h2 � (b + c)2 ⇔ a + h � b + c.

Portanto, do resultado acima e do estudo dos 3 casos, conclui-se que o mínimo da soma, AP + BP + CP é b + c,ocorrerá quando P = A.

AH

AC

HM

MC

AH

AC= = =⇔ ⇔1

22

12

cos α

BH HMMC= =

2

( )( )

– ( ) ( ) –DEFABC

= + + + + + = + =1 123

145

1645

α β γ αβ αγ βγ

( )( )

( )( )

( )( )

( )( )

DEFABC

ADEABC

BEDABC

CFEABC

+ + + =1

( )( )

( )( )

( )( )

( – ) –CFECAB

CABABC

CF

CA

EC

BC

CFEABC

CF

CA

EC

BC⋅ ⋅ ∴ ⋅ ⋅ ⋅= = = =γ β γ γ β1

( )( )

( )( )

( )( )

( – ) –BEDBCD

BCDABC

BE

BC

DB

AB

BEDABC

BE

BC

DB

AB⋅ ⋅ ∴ ⋅ ⋅ ⋅= = = =β α β β α1

( )( )

( )( )

( )( )

( – ) –ADFABF

ABFABC

AD

AB

AF

AC

ADFABC

AD

AB

AF

AC⋅ ⋅ ∴ ⋅ ⋅ ⋅= = = =α γ α α γ1

A

B H M C

α αα

A B

C

H

a

b

c

h

Page 46: obmnivel3b

SISTEMA ANGLO DE ENSINO 46 ANGLO VESTIBULARES

AULAS 5 e 6Resoluções das Atividades de Aula1. Usaremos nos itens (a) e (b) a propriedade P:

P: “os segmentos tangentes a um círculo por um ponto externo a este são congruentes”.

a) Desde que X, Y e Z são pontos de tangência dos lados BC, CA e AB respectivamente, temos:

BX

= BZ

, CX

= CY

, e AZ

= AY

[1]

BX

+ XC

= a

Como BC = a, CA = b e AB = c, têm-se: CY

+ YA

= b [2]

AZ

+ ZB

= c

Adicionando, membro a membro e em seguida agrupando convenientemente, encontramos:

(BX

+ ZB

) + (XC

+ CY

) + (AZ

+ YA

) = a + b + c [3]

Desde que 2p = a + b + c, resulta de [1], [2] e [3]:

2BX

+ 2(CY

+ YA

) = 2p ∴ BX

+ b = p ∴ BX

= p – b

Portanto, BZ

= BX

= p – b e de modo similar chegamos nas igualdades AY

= AZ

= p – a e CX

= CY

= p – c.

b) Da propriedade P resulta: EA

= AF

, FB

= BD

, DC

= CE

Daí,

Portanto, da recíproca do teorema de CEVA, conclui-se que as cevianas AD, BE e CF são concorrentes

2. Usaremos nos itens (a) e (b) a propriedade P:P: “os segmentos tangentes a um círculo por um ponto externo a este são congruentes”. a) Desde que Xb ,Yb e Zb são pontos de tangência do excírculos com os lados BC, CA e AB respectivamente,

temos: CYb

= CX b

e BXb

= BZ

b

Como BC = a, CA = b e AB = c, têm-se as igualdades:

Adicionando-se membro a membro, simplificando e agrupando convenientemente, temos:

a + (CYb

= CXb

) = b + c, ou ainda a + 2 ⋅ CXb

= b + c

Lembrando que 2p = a + b + c, então da adição de a em ambos membros, obtém-se: 2a + 2 ⋅ CYb

= 2p

Portanto, CYb

= CXb

= p – a. De modo similar, obtém-se:

b) Usando o resultado obtido em (a) têm-se:

Portanto, da recíproca do teorema de CEVA, conclui-se que as cevianas AXa, BYb e CZc são concorrentes

3. Suponhamos inicialmente que os incírculos λ1

e λ2 dos triângulos ABD e ACD respectiva-mente, (figura) sejam tangentes. Demonstrare-mos que D é o ponto de tangência do lado BCcom o incírculo do triângulo ABC, o que equi-vale provar que BD = p – b, sendo p semiperí-metro do triângulo ABCA reta AD é tangente a λ1 e λ2, conseqüente-mente, AD é tangente comum a estes incírculos.

AZ

Z B

BXX C

CY

Y A

p bp a

p cp b

p ap c

c

c

a

a

b

b

⋅ ⋅ ⋅= −−

−−

⋅ −−

=( )( )

( )( )

( )( )

1

AYB

= AZ

b = p – c

BZb

= BX

b = p

CYb

+ AYb

= b

a + CX

b = AY

b + c

AF

FB

BD

DC

CE

EA

EA

FB

FB

DC

DC

EA⋅ = ⋅ ⋅ =⋅ 1

14

24

3

N2

N1

A

u

y

u

z

CzM2M1x xDB y

T

u

Page 47: obmnivel3b

SISTEMA ANGLO DE ENSINO 47 ANGLO VESTIBULARES

no ponto T de tangência. Sejam M1 e M2 os pontos de tangência de ambos incírculos com BC. Seja N1 ponto detangência de λ1 com AB e N2 ponto de tangência de λ2, com AC, como na figura anterior.

Com estas considerações e a figura dada anteriormente, podemos escreverx = DM1 = DM2 = DT; y = BM1 = BN1; z = CM2 = CN2; u = DM1 = DM2;

conseqüentemente, dos segmentos sobre os lados do triângulo ABC, resulta:

Efetuando as operações indicadas por: (1) – (2) + (3), membro a membro, obtém-se: 2x + 2y = a – b + c.Logo 2x + 2y = a + b + c – 2b, ou seja 2x + 2y = 2p – 2b. Assim, BD = x + y = p – b.Portanto D é o ponto de tangência do incírculo do triângulo ABC com o lado BC.

4. a) Desde que D, E, F são pontos médios dos lados BC, CA e AB,e os pares de triângulos: (ADB, ABC), (BEC, ABC) e (CFA,ABC) tem a mesma altura, segue-se:

[1]

Desde que G é o baricentro do ∆ABC; e os pares de triângulos (AGB, ADB), (BGC, BEC)

e (CGA, CFA) tem a mesma altura, resulta: [2]

Logo, de [1] e [2],segue-se que:

Analogamente: (BGC) = e (CGA) = . Portanto, (AGB) = (BGC) = (CGA) = .

b) Seja L a interseção da reta MN com a tangente comumPQ. Nestas condições, decorre da potência do ponto Lem relação as circunferências C1 e C2 que:

LQ 2 = LN

⋅ LM

= LP 2

Daí, LQ

= LP

, isto é, L é ponto médio de PQ. Por outrolado, desde que L é ponto médio de PQ, têm-se sucessi-vamente:• (QLM) = (PLM) [1] (∆QLM e ∆PLM tem a mesma altura)• (QLN) = (PLN) [2] (∆QLN e ∆PLN tem a mesma altura)Como: (MNQ) = (QLM) – (QLN) e (MNP) = (PLM) – (PLN). Então, de [1] e [2]: (MNQ) = (MNP).

5. Sejam O circuncentro e I incentro do triângulo ABC. Desde que O e I são simétricos em relação ao lado AB e Oum ponto da mediatriz de AB, então I também pertence amediatriz de AB. Logo, I é eqüidistante de A e B.Daí, conclui-se que AC = CB, conseqüentemente ACB é umtriângulo isósceles. (vide gráfico)

Nestas condições, • ∠ A = ∠ B, (no triângulo ABC)

• ∆AOB e ∆AIB são triângulos congruentes. • AI, BI e CI são bissetrizes de A, B e C respectivamente.

( )ABC3

( )ABC3

( )ABC3

( )( )

( )( )

( )( )

( ) ( )AGBABC

AGBADB

ADBABC

AGAD

BDBC

AGB ABC= ⋅ = ⋅ = ⋅ = ∴ =23

12

13 3

( )( )

( )( )

( )( )

AGBADB

BGCBEC

CGACFA

= = = 23

AGAD

BGBE

CGCF

= = = 23

( )( )

( )( )

( )( )

ADBABC

BECABC

CFAABC

= = = 12

2x + y + z = a (1)z + u = b (2)y + u = c (3)

D

A

B C

FE

G

QPL

M

N

A B

C

I

M

O

N

Page 48: obmnivel3b

SISTEMA ANGLO DE ENSINO 48 ANGLO VESTIBULARES

Por outro lado, da congruência entre os triângulos AOB e AIB: ∠ AOB = ∠ AIB [1]

dos triângulos AIB e ACB: ∠ AIB = 90° + [2]

da propriedade entre ângulo central e inscrito: ∠ AOB + 2 ⋅ ∠ C = 360° [3]

Portanto, de [1] e [2]: 90° + + 2 ⋅ ∠ C = 360°, donde resulta ∠ C = 108°

6. a) Seja λ o circuncírculo de ABC. Seja H o ortocentro de ABC.Seja H’ a intersecção do prolongamento da altura CH comλ. Seja HC e HB os pés das alturas relativa aos vértices C eB respectivamente.

Nestas condições, desde que o triângulo ABC é acutângulo,segue-se que:• H é um ponto interno de ABC• HH’ é perpendicular a AB.

• ∠ ABHB = ∠ ACHC, (pois ABHB e ACHC) são ângulos agudos de triângulos retângulos ABHB e ACHC que temno vértice A um ângulo agudo comum).

• ∠ ABH’ = ∠ ACHC, (ângulos inscritos determinam em λ o mesmo arco AH’)

Portanto, destes resultados conclui-se que HHCB e H’HCB são triângulos retângulos, em H, congruentes (caso

AA). Conseqüentemente HHC

= H’HC

. Além disso, HH’ ⊥ AB

, logo H’ é o simétrico de H em relação a AB.

Portanto, como H’ pertence a λ, resulta que o simétrico de H pertence ao circuncírculo λ de ABC. (Esteresultado é verdadeiro para qualquer triângulo ABC).

b) Inicialmente liguemos os pontos A e C como também os pontos E e F por segmentos AC (diagonal) e EF. SejaN a intersecção do prolongamento do segmento EF ( de E para F) com a diagonal AC. Nestas condições, ∠ NAE= 45° ( pois AC é diagonal do quadrado) e ∠ BEF = 45°, (pois BF = BE e ∠ B = 90°). Daí, EN é perpendicular aAC; conseqüentemente EN é uma altura do triângulo ACE .Por outro lado, o lado CB é uma outra altura deste mesmo triângulo, decorrente de AB e CB serem lados de umquadrado. Assim, F, é o ortocentro deste triângulo. Portanto, AG que passa por F é uma outra altura destetriângulo, o que implica, ∠ AGE = 90°

Resoluções da Tarefa de Casa1. a) Sejam E, D e F os pontos de tangência do excírculo com o

lado AC, e os prolongamentos dos lados BC e BA conformea figura ao lado. Seja I o excentro e r o exraio.Nestas condições, o quadrilátero IDBF é um quadrado e oseu lado BD tem comprimento igual a r.

Por outro lado, (semiperímetro) e

CD—–

= CE—–

= p – BC—–

. Portanto, r = BC—–

+ CD—–

= p = 6

b) Seja ABC um triângulo retângulo em C.Seja a = CB, b = CA e c = AB.A figura ao lado mostra o incírculo do triângulo retângulo, eos pontos (X, Y e Z) de tangência com os lados. Note queIXCY é um quadrado, pois CX

—–= CY

—–e três de seus ângulos

são retos. Nestas condições, podemos escrever:

a + b = (BX—–

+ CX—–

) + (AY—–

+ CY—–

)

a + b = (BZ—–

+ CX—–

) + (AZ—–

+ CY—–

)

a + b = (BZ—–

+ AZ—–

) + (CX—–

+ CY—–

) = c + d

p = + + =3 4 52

6

∠ C2

∠ C2

H’

A

HB

HC

H

C

B

A

C DBE

Ir

rr

F

X

YC

Z

I

A

B

Page 49: obmnivel3b

SISTEMA ANGLO DE ENSINO 49 ANGLO VESTIBULARES

c) Das condições, do enunciado temos: AY—–

= p – a e AYb—–

= p – c

Como AYb—–

� AY—–

, pois BC—–

� AB—– ∴ YYb

—–= AYb

—–– AY

—–= p – c – (p – a) = a – c

2. a) Sejam A, B e C os centros das circunferências de raios a,b e c respectivamente.Seja A’, B’,C’ as projeções ortogonais de A, B e C res-pectivamente sobre a reta dada.Seja D e E as projeções ortogonais de C sobre as retasAA’ e BB’ respectivamente.Seja F a interseção da reta BB’ com a paralela a A’B’ por A.Nestas condições, os triângulos ADC, BEC, e BFA são re-tângulos em D, E e F respectivamente, conforme mostra-sena figura ao lado

Dos triângulos retângulos ADC, BEC e BFA:

Por outro lado, da figura acima:

Dividindo ambos os membros por resulta:

b) Resolução OficialA figura ao lado representa a situação, onde X e Y são ospontos médios dos segmentos MN e PQ e Z é o ponto detangência das circunferências. Então, como ∠ MBN = ∠ PDQ = 90°, segue que

BX = MX = NX = XZ e DY = QY = YP = YZ.Assim, MN + PQ = BX + XZ + ZY + YD � BD = AC.

3. Suponhamos que D seja o ponto de tangência do incírculo do triângulo ABC com BC, isto é, BD = p – b e CD = p – c.Vamos demonstrar que a reta AD é tangente em um mesmo ponto T a ambos círculos λ1 e λ2.

Inicialmente, suponhamos que :seja T1 o ponto de tangência de AD com λ1 e T2 o ponto de tangência de AD com λ2,Nomeando agora os segmentos de tangência por x1, x2, u1, u2, y, z conforme mostra na figura, têm-se:

x1 = DM1 = DT1 ; u1 = AN1 = AT1 ; y = BM1 = BN1x2 = DM2 = DT2 ; u2 = AN2 = AT2 ; z = CM2 = CN2.

Nestas condições, podemos escrever: AD = AT1 + T1 D = u1 + x1 = c – y + x1 = c + ( y + x1 ) – 2y = c + (p – b) – 2yAD = AT2 + T2 D = u2 + x2 = b – z + x2 = b + ( z + x2 ) – 2z = b + (p – c) – 2z

Destas igualdades, segue-se que: c + (p – b) – 2y = b + (p – c) – 2z ∴ z = y + b – cconseqüentemente x2 = (p – c) – z = (p – c) – (y + b – c) = (p – b) – y = x1, isto é, DT1 = DT2

Portanto, T1 = T2 = T, ou seja, AD é tangente em um mesmo ponto T a ambos círculos λ1 e λ2.Finalizando a demonstração da proposição pedida.

N2

N1

A

u1

y

u2

z

CzM2x1 x2DB y

T1T2

1 1 1

b a c+ = 2 abc

DC CE A B AF ac bc ab+ = = + =∴, , 2 2 2

DC a c a c ac CE b c b c bc e AF ab= + − = = + = =( ) – ( ) , ( ) – ( – )2 2 2 22 2 2

A Fa – bB

b – c

cC’A’ B’

a + ca + b

b + cC

a – c

D E

A

B C

D

M

N

X Z

Q

P

Page 50: obmnivel3b

SISTEMA ANGLO DE ENSINO 50 ANGLO VESTIBULARES

4. Do enunciado, temos que: AA’B e BB’A são triângulos retângulos emA’ e B’ respectivamente, com a mesma hipotenusa AB. Assim, sendoM o seu ponto médio,

[1].

Raciocinando de modo análogo para os triângulos, HA’C e HB’Cretângulos em A’ e B’ respectivamente, com a mesma hipotenusaHC e tendo N como seu ponto médio, tem-se:

[2].

Portanto, de [1] e [2], M e N são eqüidistantes de A’e B’ conseqüentemente MN é mediatriz do segmento A’B’.Nestas condições, conclui-se que MN é perpendicular a A’B’.

5. a) O círculo de diâmetro EF passa por D se, e somente se ∠ EDF = 90°. Então, problema reduz-se em provar que o

ângulo EDF é reto. Vejamos a seguir esta demonstração. Sejam G e H pontos dos prolongamentos de CA e BArespectivamente.

Logo, ∠ GAB = ∠ HAC = 180° – ∠ A = 180° – 120° = 60°. Como, ∠ DAF = ∠ DAC = 60°, conclui-se que as semi-retas AB e AC são bissetrizes dos ângulos GÂD e HÂD respectivamente.Por outro lado, do enunciado, F e E são pés das bissetrizes internas dos ângulos C e B respectivamente. Nestas con-dições , F é eqüidistante de DA e CB enquanto E é eqüidistante de DA e BC, conseqüentemente DF é bissetriz do

ângulo ADB e DE é bissetriz do ângulo ADC.

Portanto, o que finaliza a demonstração.

b) (Resolução Oficial) Desde que QT é paralelo a AB, , podemos concluir que . Portanto, QT é

paralelo a AO, o que significa que QT é base média do triângulo AOH. Então, Q é o ponto médio de AH, QP é basemédia do triângulo ARH ( pois P é o ponto médio da corda AR) e PQ e paralelo a RH. Então, RH é perpendicular a AR,donde segue que H, R e B são colineares (AR é perpendicular a RB e AB é diâmetro)

6. Duplicamos o triângulo traçando AD paralela a BC eCD paralela a BA construindo o paralelogramo ABCDcomo mostra a figura.Tomemos o comprimento do lado AB como unidade, Mcomo intersecção de CD com a reta PQ e PB = x.Então AP = 1 – x. Assim,

Por semelhança de ∆AQP e ∆QMC:

Por semelhança de ∆GPB e ∆GMD: .

Logo: MD = 2x e MC = 1 – 2x.

PBMD

GBGD

= = 12

QCQA

MCAP

MCx

= =1 –

QTAO=2

QPPO=2

∠ ∠ ∠= + = ° = °EDFADB ADC2 2

1802

90

60° 60°

60° 60°A

G H

F

B

E

D C

A N B N HC, ,= =2

A M B M AB, ,= =2

B’

N

A’ C

M

A

B

H

A

B

C

D

M

QG

P

Page 51: obmnivel3b

SISTEMA ANGLO DE ENSINO 51 ANGLO VESTIBULARES

Substituindo no primeiro membro a relação do enunciado temos:

que é uma relação verdadeira para qualquer x.

A igualdade ocorre para PQ paralela ao lado BC

AULAS 7 e 8Resoluções das Atividades de Aula1. Seja H a interseção de AE com BD.

Seja F a interseção da semi-reta CH com o lado AB.Desde que os pontos A, D, E, e B são concíclicos e ∠ AEB = 90º, então∠ ADB= 90º (a circunferência que “passa” por estes pontos tem AB comodiâmetro). Daí, AE, BD, são alturas de ABC. Então, H é ortocentro e CF aoutra altura, do triângulo ABC. Nestas condições, os ângulos FHB e DHC são opostos pelo vértice H, BFHé um triângulo retângulo em F e HECD é um quadrilátero cíclico

(∠ CEA = ∠ CEH = 90º e ∠ BDC = ∠ HDC = 90º).Conseqüentemente, podemos escrever:

x = ∠ CED = ∠ CHD = ∠ FHB = 90º – 20º = 70º. Portanto, x = 70º.

2. primeira resoluçãoNas condições do enunciado o quadrilátero ABCM é cíclico. Então aplicando o teorema de Ptolomeu, têm-seAM— ⋅ BC

—= MB

— ⋅ AC—

+ CM— ⋅ AB

—. Desde que o triângulo ABC é eqüilátero têm-se: BC

—= AC

—= AB

—resulta da

igualdade acima AM—

= BM—

+ CM—

, o que finaliza a demonstração.

Uma segunda resoluçãoFazendo uma rotação de 60º em torno de A no sentido anti-horário. O ponto B transforma-se no ponto B’ (B’ = C) e o ponto M em M’ com ∠ BAM = ∠ CAM’.Então, o segmento AB transforma-se no segmento AC enquanto que o segmento AM transforma-se no segmento AM’,conforme vemos na figura abaixo:

Como conseqüência desta rotação, triângulo ACM’ é congruente ao triângulo ABM com os pontos M, C e M’ ali-nhados nesta ordem. Nestas condições, podemos afirmar que o triângulo AMM’ é eqüilátero de lado AM e o segmentoCM’ é congruente ao segmento BM, conseqüentemente, AM

—= BM

—+ CM

—, o que finaliza a demonstração.

3. Seja ∠ ABD = α.

Desde que AB // DC, então ∠ ABD = ∠ BDC = α. (a. i.)

Por outro lado, senα = (Do ∆ABD) e cosα = (Do ∆ABD)

Daí,

A igualdade ocorre quando sen2α = 1, ou seja, quando α = 45º.

Portanto, o valor mínimo de é 2.

CD

AD

ADCD

ADBD

BDCD

sen sen CDAD sen

= = = =⋅ ⋅ ∴α αα

αcos 22

22

2�

BD

CD

AD

BD

A

B

MC = B’

M’

PB x MC= = =⇔ ⇔13

13

PBPA

QCQA

x x

xx x x⋅ ⇔ ⇔ + ⇔� � � �

14

1 2

1

14

9 6 1 0 3 1 02

2 2( – )

( – )– ( – )

B CE

DF

A

Hx x

x

BA

D C

α

α

Page 52: obmnivel3b

SISTEMA ANGLO DE ENSINO 52 ANGLO VESTIBULARES

4. a) Seja E a interseção da reta perpendicular ao lado AD por T.Sejam α = ∠ BTP e β = ∠ PTC

Desde que BD é perpendicular a AC, então:∠ BTC = ∠ BTP + ∠ PTC = α + β = 90º.

Desde que T é a interseção de BD e EP, temos:∠ DTE = ∠ BTP = α (opostos pelo vértice)∠ ETA = ∠ PTC = β (opostos pelo vértice)

Desde que TE ⊥ AD e BD ⊥ AC, temos que

∠ DTE = ∠ TAE = α (ETA complementa ambos os ângulos) .

∠ ETA = ∠ EDT = β (DTE complementa ambos os ângulos).

Nestas condições, têm-se da propriedade: “Se dois ângulos inscritos em uma mesma circunferência “enxer-gam” o mesmo arco nesta, então eles são congruentes” , que:

∠ PBT = ∠ CBD = ∠ CAD = ∠ TAE = ∠ DTE = ∠ BTP = α∠ TCP = ∠ ACB = ∠ ADB = ∠ EDT = ∠ ETA = ∠ PTC = β

Portanto, destas duas últimas igualdades, conclui-se que os triângulos PTB (PB = PT) e PCT (PC = PT) sãoisósceles. Daí, resulta PC

—= PT

—= PB

—. Conseqüentemente P é ponto médio de BC, isto é, a reta através de T e

perpendicular ao lado AD contém o ponto P médio de BC, finalizando a demonstração pedida.

b) Seja M ponto médio de AD e P ponto médio de BC.Desde que O é o circuncentro de ABCD, têm-se que OM e OP sãomediatrizes de AD e BC respectivamente. Daí,

OM ⊥ AD e OP ⊥ BC [1]

Desde que M é ponto médio de AD e BD ⊥ AC, segue-se que ATD étriângulo retângulo em T e TM a mediana relativa a hipotenusa AD.Daí, MT = MA = MD = (1/2) AD [2]

Desde que TE ⊥ AD, então TP ⊥ AD [3]

Por outro lado, construindo por T uma perpendicular ao lado BC echamando de N a interseção desta reta com este lado, resulta peloteorema demonstrado na parte (a) desta questão, que e a reta TNcontém o ponto M. E conseqüentemente MT ⊥ BC. [4]

Por outro lado,De [1] e [3], obtém-se OM e TP perpendiculares a AD, logo MO // TPDe [1] e [4], obtém-se OP e MT perpendiculares a BC, logo OP // MT

Nestas condições, OPTM é um paralelogramo. Além disso, de [2], segue-se que OP—

= MT—

= (1/2) AD—

. O que fina-liza a demonstração.

Nota: Na figura, desde que O é o circuncentro de ABCD, então os triângulos OBC e ODA são isósceles comOB—

= OC—

= OD—

= OA—

= R (circunraio de ABCD). Daí, OPC e DOM, são triângulos retângulo, em P e M, respec-tivamente.

5. (Treinamento Olímpico Brasileiro) Seja AD a bissetriz do ângulo A do triângulo ABC. Sejam M e N os pontossobre as semi-retas AB e AC respectivamente e P o ponto de interseção de AD com MN. Prove que:

Se ∠ ADM = ∠ CBA e ∠ NDA = ∠ ACB, então AD—3 = AB

— ⋅ AC— ⋅ AP

Sejam ∠ ADM = ∠ CBA = β, ∠ NDA = ∠ ACB = α e ∠ BAC = 2γDo triângulo ABC: 2γ + α + β = 180º. Logo,

∠ NDM + ∠ BAC = ∠ NDA + ∠ ADM + ∠ BAC

= α + β + 2γ= 180º.

A

D

B C

β

β

α

α

E

T

O

P

β

α

T

R

RO

A

D

EM

N PBC

Page 53: obmnivel3b

SISTEMA ANGLO DE ENSINO 53 ANGLO VESTIBULARES

Daí, conclui-se que MDNA é um quadrilátero cíclico.Conseqüentemente

∠ CBA = ∠ ADM = ∠ ANM = β∠ ACB = ∠ NDA = ∠ NMA = α.

Além disso, sendo AD bissetriz do BÂC, tem-se também, DM—

= DN—

, eportanto o triângulo MDN é isósceles, com

∠ MND = ∠ PND = ∠ PMD = ∠ NMD = γ,

Nestas condições, observando atentamente os ângulos da figura, des-cobrimos três pares de triângulos:

(∆ADM, ∆ABD), (∆ADN, ∆ACD) e (∆AMD, ∆APN),

semelhantes. (∼ AA) ( verifique !!)

Por outro lado, destas semelhanças, encontramos respectivamente:

Multiplicando membro a membro, resulta das igualdades acima:

Portanto, simplificando, resulta da igualdade acima: o que finaliza a demonstração pedida.

Resoluções da Tarefa de Casa

1. Aplicando o teorema de Ptolomeu, Como ABC é eqüilátero, segue-se que:

Por outro lado, note que os triângulos APC e BPD são semelhantes, pois ∠ PAC = ∠ PBD, e

∠ APC = ∠ BPD = 60º. Assim, desta semelhança, têm-se, substituindo os dados do enunciado encon-

tra-se donde conclui-se que AD—

= 21.

2. a) Lembremos que um quadrilátero convexo ABCD é trapézio, se um parde lados deste quadrilátero são paralelos. Para provar a proposiçãopedida, mostraremos que AB = OD, AD = CO e ∠ BAC =∠ BDA (hi-pótese) implicam necessariamente na existência de um par de ladosparalelos.

Demonstração:

Desde que é BÔA externo para o triângulo AOD, temos:∠ BOA = ∠ OAD + ∠ ODA = ∠ OAD + ∠ OAB = ∠ BAD

Unindo ao resultado acima a hipótese CO = AD e AB = DO conclui-se queos triângulos BAD e DOC são congruentes (LAL). Conseqüentemente∠ ABD = ∠ ODC, o que implica necessariamente que os lados AB e CDsejam paralelos, isto é, que o quadrilátero ABCD seja um trapézio. O quefinaliza a demonstração.

2520

5

25=

–,

AD

AP

PC

BP

PD= ,

AP AB AC= + = 25.

AP BC AB CP AC BP⋅ = +⋅ ⋅ .

AD AB AC AP3

= ⋅ ⋅

AD AD AD AD AM AN AB AC AP AM AN AD⋅ ⋅ ⋅ ⋅ ⋅ ⋅ ⋅ ⋅ ⋅ ⋅=

AM

AP

AD

ANAM AN AP AD= = ⋅∴ ⋅

AN

AD

AD

ACAD AD AC AN= = ⋅∴ ⋅

AM

AD

AD

ABAD AD AB AM= = ⋅∴ ⋅

A

γ γ

βα γβ

αB C

D

P

M

N

γα

β

A

D

O

B

C

Page 54: obmnivel3b

SISTEMA ANGLO DE ENSINO 54 ANGLO VESTIBULARES

b) Seja E a interseção dos prolongamentos de PD e CB.Desde que P é ponto médio do lado AB do retângulo ABCD, entãoos triângulos retângulos DPA e EPB são congruentes (ALA). Daí,BE—

= AD—

= BC—

. Logo, B é ponto médio de CE. ConseqüentementeBQ é a mediana relativa a hipotenusa CE do triângulo retânguloCQE, em Q. Portanto, BQ

—= BC—

= (1/2) CE—

, o que implica dizer que otriângulo BQC é isósceles.

c) Tracemos um segmento DE com E sobre AB e ∠ AED = ∠ ABC = α.Em seguida, marcamos sobre ele o ponto F, onde DF

—= FE

—. Nes-

tas condições, decorre do enunciado:

Por outro lado, nomeando por β e θ as medidas dos ângulosAFD e ADF respectivamente, por γ a do ângulo FÂE, obtemosdo resultado acima, a figura auxiliar ao lado.

Nestas condições, decorre do enunciado e da figura auxiliar acima:θ + α + β + γ = 180º. (∆ADE) [1]α + β + γ = 120º. (enunciado) [2]β = α + γ (ângulo externo do ∆AFE ) [3]

De [1], [2] e [3] resulta β = θ = 60º e α = γ = 30º. Portanto, ∠ DAB = γ + β = 90º.Resposta: O ângulo DÂB mede 90º.

3. a) Decorre do enunciado que: A1P // B1Q.

Daí, têm-se que A1PQB1 é um trapézio isósceles e que suas diagonais tem o mesmo comprimento, conseqüen-

temente . Por outro lado, ∠ PA1Q = ∠ PB1Q pois determinam o mesmo arco PQ em S1 e de modo

análogo temos de S2, ∠ PA2Q = ∠ PB2Q. Nestas condições, os triângulos A1QA2 e B1PB2 são semelhantes,

mas como , conclui-se que os triângulos são congruentes, conseqüentemente tem o mesmoperímetro.

b) Sendo ∠ BAL = ∠ CDK segue-se que o segmento KL é enxergado pelos vérti-ces A e D do trapézio por ângulos de mesma medida. Nestas condições, A,D, L e K são concíclicos; conseqüentemente ADLK é um quadriláterocíclico.Assim, ∠ BKL + ∠ BCL = (180º – ∠ AKL) + (180º – ∠ ADL) = 180º.Como conseqüência conclui-se que BCKL é também cíclico.Nestas condições, podemos escrever ∠ ABL = ∠ DCK

e ∠ BLA = 180º – ∠ ABL – ∠ BAL

= 180° – ∠ DCK – ∠ CDK

= ∠ CKD. o que finaliza a demonstração.

4. a) Sejam ABCD o quadrilátero, M, N, P e Q os pontos médios dos lados AB, BC, CD e DA, respectivamente. MNe PQ são paralelos à diagonal AC e medem a metade de seu comprimento, enquanto NP e QM são paralelos àdiagonal BD e medem a metade de seu comprimento. Assim, MNPQ é um paralelogramo. As condições doproblema dizem que a reta que passa pelos pontos médios de dois lados opostos de ABCD (digamos MP

—, sem

perda de generalidade) formam ângulos iguais com AC—

e BD—

, portanto com PQ—

e NP—

, donde MP é bissetriz deNPQ. Logo MNPQ deve ser um losango, donde MN

—= NP

—, e portanto AC

—= BD

—(pois MN

—= AC

—/2 e NP

—= BD

—/2).

b) Sejam M e N os pontos médios dos lados AC e BD respectivamente.Sejam P e Q os pontos médios dos lados AB e CD respectivamente.

A Q B P1 1=

A Q B Q1 1=

DF FE DE BC x= = = =⋅ ⋅12

12

A B

CD

QP

E

A

D C

F

x

x

2x

E Bααγ

β

β

θ

x

B C

LK

A D

Page 55: obmnivel3b

SISTEMA ANGLO DE ENSINO 55 ANGLO VESTIBULARES

Desde que AD—

= BC—

, BC não é paralela a AD, pois caso contrário ABCD seria um paralelogramo e conseqüen-temente. M = N, o que contraria o enunciado.

Nestas condições, os segmentos PM, MQ, QN e PN são respectivamente bases médias dos triângulos ABC, CAD,DBC e BAD, segue-se:

mas como AD = BC, conclui-se de [1] e [2] que PNQM é umparalelogramo de lados iguais, isto é, losango e MN é uma desuas diagonais. Conseqüentemente, MN é bissetriz do ângulosinterno PNQ. Portanto forma ângulos iguais com PN e NQ, mascomo PM//AB e NQ//BC, conclui-se que a reta MN forma ân-gulos iguais com AD e BC, finalizando a demonstração pedida.

5. Seja K a intersecção das bissetrizes relativas a E e F. (figura abaixo). Por outro lado, do quadrilátero cíclicoABCD, e dos triângulos ABF e ADE, segue-se respectivamente:

Destas igualdades, obtém-se: [1]

Por outro lado, dos triângulos FKE e FCE, resulta respectivamente:

De [1], [2] e [3] podemos escrever: ∠ K + (90° – ∠ A) + (180° – ∠ C) = 180°

Lembrando que ∠ A + ∠ C = 180°, conclui-se da igualdade acima, ∠ K = 90°Portanto, este último resultado finaliza a demonstração da proposição pedida: “As bissetrizes por E e F sãoperpendiculares”.

6. Sejam os pontos L, M e N médios dos segmentos BP, PQ e BQ respectivamente, como mostra-se na figura aolado. Assim, os segmentos NM e LM são bases média do triângulo PQB, o que implica:

[1]

De MN // PL e MN // QN, resulta, ∠ PLM = ∠ MNQ. [2]

Por outro lado, desde que ABCD é cíclico, tem-se:∠ A + ∠ C = 180°. Então ∠ C = 90°.

Além disso, como AP—

= AB—

e BC—

= CQ—

, segue-se que:• BAP é um triângulo retângulo em A e isósceles, conseqüentemente e

AL—

= LP—

= LB—

e ∠ ALP = ∠ LBA + ∠ LAB = 45° + 45° = 90°. [3]

• BCQ é um triângulo isósceles e retângulo em C, conseqüentemente —CN =

—NQ =

—NB e ∠ CNQ = ∠ NBC + ∠ NCB = 45° + 45° = 90°. [4]

MN PL e MN PL PB

ML QN e ML QN QB

//

//

= =

= =

1212

∠ ∠ ∠ ∠

∠ ∠

+ ⋅ +

+ ⋅ + ∠

= °

+ + ∠ = °

K F CFE E CEF

C CFE CEF

12

12

180

180

[2]

[3]

12

90⋅ + = ° −∠ ∠ ∠( )F E A

∠ ∠ ∠ ∠

⋅ ∠ ⋅ ∠

⋅ ∠ ⋅ ∠

+ = + = °

= ° + ∠

= ° + ∠

A C B D

F A B

E A D

18012

9012

12

9012

– ( )

– ( )

NQ PM BC e NQ PM BC// // ( / )= = 1 2 [2]

MQ PN AD e MQ PN AD// // ( / )= = 1 2 [1]

A

B

P

N

M

C

Q

D

α

α α

α

F

E

B

C

K

A

D

D

A

PQ

M

B

CL N

Page 56: obmnivel3b

SISTEMA ANGLO DE ENSINO 56 ANGLO VESTIBULARES

Nestas condições, de [1], [2] ,[3] e [4] os triângulos ALM e MNC são congruentes (LAL), pois:

Portanto, da congruência acima, conclui-se que os segmentos AM e CM são congruentes, o que equivale dizerque M é eqüidistante de A e C, isto é, C pertence a mediatriz de AC; finalizando a demonstração da proposiçãopedida.

Agradecimentos

aos professores Alexandre Marmo e Rosso (professores do Anglo SP), e Cláudio Arconcher (Anglo de Jundiai) eFrancisco de Castro (Anglo de Santos), por sugestões de exercícios e idéias para a resolução de vários proble-mas que foram de grande importância para este trabalho.

AL—

= MN—

e LM—

= NC—

∠ ALM = 90° + ∠ PLM = 90° + ∠ MNQ = ∠ MNC